You are on page 1of 343
yoursmahboob.wordpress.com Theory & Problems With Explanations OPES & Aptitude Rae STAB) SCs cud Sed Sey Sa a (Geer ae > ~~ ‘Scanned by CamScanner ‘the chapter of number system is amongst the most important chapter in the whole of Mathematics syllabus for Competitive Examinations: we can consider this chapter as the backbone trmathematics. The students are advised G0 through this chapter with utmost care; ipgeretanding each ard every aspect ofthis op. inthis chapter basic definition of itferentiypes gfnumbers are given. Then on the Basis of tis, various questions with solved examples are gen for sasy understanding: Number System Natural Numbers Counting numbers 4, 2, 9, 4, 8 sesr-vree OFS known as natural numbers. The set of natural numioers can be reprosented byN= (1, 2.3, 4) 5 nd Whole Numbers iiwe include © among the natural numbers, then thenumbers 0, 1, 2.3, 4. Sete. are called whole numbers. Whole number W=(0, 4, 2, 3,4, 5-4 Ceaty, every netural number is a whole number fut0is@ whole number which is not a neterel umber, San cesar MADE EASY oursmahboob.wordpress.com Number System Integers ‘Ail counting numbers and their negatives including zero are known as integers. The sot of integers 8, -2, 1,0, 1,.2.3. 4.1 Positive Integers The sat (1, 2.3, 4, 5... 18 a sat of all positive integers Positive Integars end natural numbers are synonyms. Negative Integers The set [-1, 2-3, ...n) Is. @ set of all negative integers. 0 is neither positive nor negative Rational Numbers The numbers of the form 9/9, where p and q are integers and q#0are known es ational numoets- 25 4 og: BF. g oe Irrational Numbers Those numbers which when expressedin decimal form are neither terminating ner repeating decimals, are known as rational numbers eg. V2,¥8. V5. rete: 2 2 Note: That the exect yaluo ot nis not = ® rational number while js_an irrational number _——— et . Reasoning & Aptitude is yoursmuathboob.wordpress.com,, to 4 equivalent aso consider somo times # this 1% ive and note exact vl ; Soniary 214i not an oxact velUo O valua is also aporoximato value ot Roal Numbers ‘The rational and irrational num! togathor are called real numbers. cs all real 69. Hi. 4 + 12,645 atc. are umbers. The sot of all real numbers are denoted by R. bers combined Complex Numbers Complex numbers can be represented in form of ‘a ib, where a and b are real numbers and i= V1. Thus 3+4i, 6+2i, |, 21 ete aro Complex numbers: Even Numbers Allthose numbers which are exactly divisible by 2erecalied aven numbers. 09.2, 4,6, 8, 10, etc. (Odd Numbers All those numbers which are not exactly divisible by 2are called odd number. Fo. 13,5, 7,9.te Prime Numbers Number civisible by 1 and itself and not divisible Py any other number aro called prime numb. a &3..2.3,5,7, 11 etc, Ne: 2 fs the only Prime number why ich Is Fest all aro oda Prime numbers wen. Composite Numbers, Natural numbers. Gree ater than 1 which Prime are composite numberg mene 90:4, 6,8, 15, otc Co prime Numbory 1 | Ronsoning & Apttuce Scanned by CamScanner ag.: (3, 748, 9) (36, 25) ate Noto: Here 8 & 9.ara N04 PIM ran, they have only one commen fac; So thoy are coprime numbers (6. 18)are coprime numbers decays two common factors io. 1 & 3, bis hon. Test of Divisibility 1. Divisibility by 2 A number is divisible by 2 if tha yp 2070 OF divisible by 2. 09. 22, 42, 84, 3872 etc. 2. Divisibility by3 Annumber is divisible by 3il the sum 9 in the number is divisible by 3 0g.: 2553 Hore 245 +5 +3.= 15, whichis divisbingy S hence 2563 is divisible by 3 3. Divisibility by 4 A number is divisible by 4 if ts last two digt 's divisible by 4, 9g.: 2652, hore 52 is divisible by 4 so 2652 is divisible by 4 09.: 3772, 584, 904 etc 4. Divisibility by number is divisible by 5 if the units digit Number is 0 or 5, @9.: 50, 505, 405 ote 5. Divisibility by 6 ‘A number Is divisible by 6 it tho numibe even and sum of digits is divisible by 3 €9.: 4536 is an even number also sum of dit 44843468 18 is divisible by 5. 99: 72, 8448, 3972 ote 8. Divisibitty by 8 A numbor is divisible by 8 it last vee di! OF Its divisible by 8 0G: 47472 hore 472 is divisiblo by 6 1 {nls number 47472 is divisibla by 8 Wg ign ape tA wocesoursmahboob.wordpress.com 7 Owisibity by 9 AMUMbEF IS ding: 31S Ovsdie bys F 198936 Nee 10.8.9. S dwisDie oy +5 0 Y 98 hence 10896 72 te sum of as © « 8 Divisibuity by 19 A number is duis bie By 10 sO €9.° 90, 900, 740, 34920 ate 9. Divisibility by 11 A number '$ divisible by 11 i the aterence of sum Of 394 at Odd places ang sum of Gos at even Places 's ether Dor divs bie by €9.: 1331, the sum of dig at odd piace s J+3 and sum of aigt at even places is 3+ and Der dtterence s 4-4 =0 $0 13315 f ts una age owsibie by 11 1. (asof =a? +220 +0% HCE and LCM of Numbers 2. (a-vf =a? - 200-0? (Hignest Common Factor) ot waormorenumber «3. (a-b)(a+b)=a’—0? Is the greatest number that divides each one of them exactly. For example 8 is the highest common factor of 16 ana 40 HCF is also called greatest common divisor (G CD) 0G CM ic. Greatest Common Treasure 4. (arof +(a-p}’ =2{2? +0") 5. (aspf -(a-py = aap 6. (a+d)’ =a" +3e°0+3an* +0? +0" +3adla+d) -3a°d+San*-0° LOM (Least Common Mulupie) of two or more number 's the least or a lowest number which is exacty Gnisibie by each of them. For example LOMO! Band 12:524 pecauset's = g 33.05 (agnj(a? -aoed') the first number which is multiple of bot 6 ang 12 LCM and HCF of Fractions Numerator a abso' +c? 3200 Fractions are written in form Of Bec ominator 7. (a-op 0? -3ap(a-5) 9, a°—b? =(a-b)(a? ++") Yogi ep +c? -ap-be-ca " e(aroec! a asin Ape MADE EASY Scanned by CamScanner " yoursmahboob.wordpress.com. eee" ear oye-P) ty tor Alzabraic Fu ynction conditon of Duis ty divisible by a4 only 1, an bP is oxaal when nis ocd exe? 4b? = (040) b+ 2. ate bris never divisible DY niscodd or even) Exe b9= (a+ je by (aD) a is also not aivisibte by (a~ D) pr is exactly divisible by (a-b) ether ris odd or even) cei bi =(a-b)(e+b} cots dMisible bya-b: a3—b? = (a - bla’ + ab + b%) 80 it is aivisicle by (a~b); af bts (22) ~ (04) = (02 - b?) (a? + b) = (aja + b) (a? + b4) s0 itis dNisible by (a-b) Simianly . ? is exactly divisible by (a—b) also al} is also exactly dvsioio by (a—b). 42 -ap+b#) scbvsble jg divisible by &*P +b, also bya +0, als eenarar py (et -ab + BA) srt FACTORS OF COMPOSITE NUI MBER Composite numoers are t umbers which are not pr Forex: 8 isacom ID Osi » fecsea no He RU Sinc it can be ime, Composite number 10 10 Note: + 2 MADE. $B Note: 17's aprime number and all prime number have two factors 4 & itself ° Ex.8. Find all the factors of 7, Sol.: 47 = 47" So factors =(1+1)=2 Counting Number of Zeros Sometimes we coms across problems in which we have to count number of zeros at the end of factorial of any number. For example Number of zero at the end of 10! 10! = 10 x9 x8X7 x6 x5 x4 x9x2x1 Here basically we have to count number of fives, because multiplication of five by any even number will resultin Oat the end of inal product. In 19! we have 2 fives thus total number of zeros are 2. Short Cut: Counting number of zeros at the end of n! a nn Value willbe ¢+ E+ as +a The integral value of this sum will be the total number of zeros. Ex. 1 Number of zeros at the end ot 10! 10 10 ete here intagral value is 2. Note: Here z is less than 1 50 will not count it. Ex. 2 Number of zeros at the end of 100! oursmahbhoob.WOrdpress.6RQMpin — +s Ex3 Number of zeros at the end of 126) 126 = integral value will be 2548 +1 =31ze108 Sol.: x4 Number of zeros at ne end of 500)" 500 , 500 , $00 , 500 Sol: > sete tet integral value wil be 100+ 20+4 = 12426108, ES a of zeres at the end of 1000! . 41000 , 1000 , 1000, 1000 Sol. —" > “ete tet 200 + 40.48 +1 = 249 ze108 Ex.6 Number of zeros at the end of 50! oa: es Integral value will be 10 +2= 12 zeros Sol.: Ex7 Number of zeros at the end of 90! Integral value will be 18 +3 = 21 zeros, Cyclicity Cyclicity of a number is used mainly for the calculation of unit digits. 1. Cyelicity of 1 In 1®, unit digit will aways be 1 2. Cyelicity of 2. Sol.: cy ey integral value will be wae 20 +4 = 24 z0r08 oa @ Reasoning & Aptitude 13 MADE EASY Scanned by CamScanner voursmahboob.wordpress.cain, 1 P= 256 Atter every fourth interval 2, 4,8, 6 arerepoated socycie of 2is 2, 4. 8, 6. Ext Find unit digit of number 2 Sol; Hero unt digt will repeat a8 2.4, 8. 6 after every four interval til 82 be 2 and 4 will be 4. So unit dight will be 4 x2 Find unit digit of 2. Sol; Here2, 4,8, Gwill repeat ater every four intorval ti! 320 nex! digit willbe 2, 4, [3]. So unit digit of 2° will be 8. x3 Find unit digitof 12"? x22 Sol: Unit digit of 12"? will be 6 and 22 will be 4. So unit digit of 127? x22” will be 6x4 = 2 [a]; 4 Ans, 3. Cyolicity of 3. After every fourth interval 3,9, 7 and 1 are repeated. So cycle of Sis 3,9, 7, 1. Ext Find unit digit of a8, Sol: Cycle of3is3,9, 7, 1 which repeats after every four interval til 3®, So next unit digit will be 3. Ex2 Find unit digitof 133988, Sol: Cycle ofis3,9, 7,1 which repeats atter ‘every fourth interval till 133'%2. go next unit digit wil be 3, “ Reasoning & Aptitude Scanned by CamScanner x3 Find unit digit of 963° x73” Sol. Unitdigit of 963% will be 7 ang, of 7379 will Be 3. So unit qa” i 9635 x7372 will be7 x3.= 21 16 $7 eg x4 Find unit digit of 122" x 13g Sol. Unit digt of 122" will be 4 be, cycle of 2 is 2, 4, 8, € and unt gant 193193 will be 8. So unit gan 42212 x133"%3 will be 4x3 — a < 2 Ans. _ Cyellcity of 4. aed B= 16 = 64 44 = 256 Cycle is 4, 6, ie. Unit digit of 4" depends on value of n If nis odd unit digit is 4 andif niseven digt is 6. Ex.1 Find unit digit of 44°. Sol |: Since 425 is odd number unit digit wil be 4. Ex.2 Find unit digit of 410. Sol. Since 1024 is even number unit digitwil beé. Ex.3 Find unit digit of 133° x4”, Sol. 5. + Unit digit of 1336 is 7 and unit cig 449 is 4 50 unit digit of 133° x4" willbe 7x4 =28i.e.8. Cyclicity of 5. Unit digit will always be 5. Cyclicity of 6. 8-6 =36 @= 216 6 = 1296 Untt digit will always be 6 waver" ee ———-yoursmahbeoebaverdpress eon | Ex.1 Find unit digit of 4% x g® Sol Unit digit of 4° ig 4 and unit digit of Sis 6 so unit digit of 4” x6° willbe 4 x6 = 24 ie. 4, Ex.2 Find unit digit of 14419 126126 Sol.: Unit digit of 144% is 4 and unit digit of 126" is 6. So unit digit of 1447 x 126126 will be 4x6 = 2416. 4, 7. Cyelicity of 7. 78 = 5764801 Cycle of 7 is 7,9, 3,1 Ex. Find unit digit of 1777 Sol. Cycle of 7 repeats 7, 9, 3, 1 till 17° next digit will be 7. So ans. is 7. Ex2 Find unit digit of 177 x27?” Sol.: Unit digit of 17” is 7 and unit digit of 2777 is 3. So unit digit of 1717 x 2777 will be7x3=2tie.1 Ex.3 Find unit digit of 1777 x27 «377 Sol: Unit digit of 1717 is 7, unit digit of 27°7 is 3 and unit digit of 372” is 7. So unit digit of 1717 «27°? x37 will be 7 x3 x7 = 147 ie. 7. 8. Cyclicity of 8. B'=8 B= 64 = 512 64 = 4096 96 = 32768 So cycle of 8 is 8, 4. 2. 6. 18 Find unit digit of 18! 50 ; Cycle of 8 repeats & 4 26 after every MADE EASY Scanned by CamScanner four intervals til 18" next digits will be 8 8nd 4. So unit digit of 18" will be 4 Ex.2 Find unit digit of 18° x28 x2g97* Sol: Unit digit of 18'8 is 4, unit digit of 267% is 6, unit digit of 288° is 6, So unit digit of 181° x28 x 286% will be 4 x6 x6 = 144 ie. 4. 9. Cyolicity of 9. gizo 92-81 #=729 94 = 6561 Cycle of 9is 9, 1 {In & unit cigit will be 9 if n is odd and unit digit will be 1 if nis even Ex.1 Find unit digit of 11774 1212 + 1319 4 14144 1515 Sol: Unit digit of 11" is 1 Unit digit of 12°? is 6 Unit digit of 13°? is 3 Unit digit of 14°* is 6 Unit digit of 1515 is 5 ‘So unit digit of given sum will be 1464+34+6+5=21ie1 Ex. 2 Find unit digit of 217 x22 x23% x 247 x 259 Sol; 25% will give 5 in unit place, when multipied by any even number .e.0,2,4, 6, 8 itwil give zero at unt place. So, zero will be the unt digit of gwven question CYCLICITY TABLE eal Ne ao pon aos ore) ne oo COBVaneens Sepnon 15 @ Reasoning & Aptitude | { | 16 | © Number Syatom Remainder Theorem andre ig gxoxe Remainder of exoression 4 [ie a xb x: wien divided by nf equal tthe remainder of orpreecon #8 #5 9, g, x0, 5, when Sided ky n], whera ais remainder when a is divided by n, b, ie romaindor whon b is divided by n, and Gls remainder when c Is divided by 1. x1. Find the remainder of 18 « 17 19 when divided by 7, 181719 Sol: Remainder of exproasion 19 iy) eeaunic HEE 16. 1 stet les Oncividing 1Sby 7 waget as remainder Ondividing 17 by? we get 32s remainder Ondividing 19 by 7weget as remainder {nd combined remainder wil be equel to 5 remainder of 1 jg, 4 2 omar presen 1x02 will be equal t remainder of exression 2x3 Ta Which is equal tn 6. Oooo Polynomial Theorem THs ety Povorhecusn nd he reneindar ‘According to polynomial theorem, (esa erenc, wt Scanned by CamScanner yoursmahboob.wordpress.com- Moor, ere x7 #nCy x al + ny v? ge, cea enact] ¥ EL remainder of expression (2) wil bg eau remainder of < because reat of thy es contains x are completely divisibie by « cu eH a Aecerding to pclyncmial theoren remainder will be equal to reriaince 2 te expression whichiseqal int ow 2 Findremainder ot sa: © 7+" ; 72 Fie Ex Fd emainder of M1317 Watt? | Sets _ Sol: SRBAIT 2, See tot 6 768 76 According to remainder theore.] x4 Find remainder of 2 7 gi 42] 0 oq Bazi 2m 7 7 7 x2 = Ex? PP ne 7 2) bes Eke wxgursmahboob.wordpress.com 7 . 10 6 Find emaincer of 5 BO [3x74ay _ (aye 7 7 fo) a9 Px 77 = Remainder is 2. Ex.7 Find remainder of 3 ey any “= aT 7 7 7 7 wp 22 Tax tx 7 7 7 = Remainder is 2 0 Ex Find romaindor of ©. Sol, - Rumaindor is 4 MADE EASY Scanned by CamScanner © Number System ) 17 MORE ON REMAINDERS Case-4 On dividing a numbor by a, b& e it we get @k, DK and ck as remainder respoctivly then that number willbe n + LOMot fa, b,c] For ex. 1: On dvlding a number by 4.58.6 we 9613, 4,8 5 as remainder Find te nurioer Sol 45 6 Renander ggg Which is equal to (4= 1), 6-1) 6-1) 60 that number wi be ne LoMet (4 5,5)-—1 ie, 60-1259 Note: nsucnnumivers are possible, Here worave taken mas 1, Other numpere aro 11, 170,208, et Where value of nis 2,3, & 4 respectively Ex.2 On dividing a number by 5, 6 and 7 ne got 3, 4 and § as remainder. Find the ‘number, Sol, 5 8 Tf Remainder 304 «5 which is equal 10 (6-2), (6-2), (/-2) that number will be LOM of (5,6, 7)~2= 210-2 = 208, Note: Here we have taken value of n as 1 Ex3 On dividing a number by 4 and 5 we get 2&3 as remainders. Find the highest double digt such numbe Sol. 48 Ramainder a 3 which isequal to (4-2), (9-2) that number wil be: neLCMot 4, 5]-2 We get 18, 26, 58, 78, 98, 118 when value ol nis 1, 2.3, 4, 5 and 6 respectively Highost double digit such number will De 98 Ans. w © Reasoning & Aptitude | ye |» Num xa Oncividing @ rumber by 4, 5 and 6 we get 2, 3 and 4s remainder tind highest poesibe three digit such number ° 4 5 86 Remainder 2 4 4 which is equelto (4-2), (6-2 (6-2) that number will be: neLcMol (4, 5, 6-2=n x60-2 When n = 1 we get 58. Highest possible three digitsuch number will be 958. x5 On dividing a number by 5, 8 and 7 we (get 3, 4 and 5 as remainder. Find highest possible three digit such number. Sol. 5 6 7 Remainder 3 4 5 which is equal to (5-2), (6-2), (7 - 2) that number will be: n#LOM (5, 6, 7)-2=n x210-2 Highest possible three digit number will be 838. Case-tl (On dividing a number by a, b and cif we get k as remainder always, then that number will be n*LCMot (a, b, c)+k. Ex.1 On dividing a number by 5, 6 and 7 it we get Zasremiander always, find that num | ber That number will de ne LOMof (5, 8, 7]+2 210 +2 =212 On dividing a number by 6 and 7 we get 4a remainder always, find the highest __ Possible tee digit such number Sol: n*LCM[6, 7] +4 =1*424 4; tree digit highest possible such num 966442970 Ans em be 18 Reasoning & Aptitude Scanned by CamScanner yoursmahboob.wordpress.cbm Case-Ill ra number after adding kis exactly, bya, band c then that numbor will by iii n#LCM (a, b, clk Ext Find a number which after adgin, divisible by 6 and 7. Sol: That number will be ne LCMof (6, 7/5 itn = 1 then 42-6 =37. 55 Ex.2. Find a number which after adding 7 , divisible by 10, 11 and 12, . That number will be ne LOMof[10, 11, 12]-7 ifn= 1 then 660 - 7 = 653 Ans. Squares of Numbers Squares of numbers are frequently used fo calculations on various types of problems. It advisable to remember square of at least fist thirty numbers 12 = 121 122 = 144 1 1a 16? = 256 172 = 269 1-24 From following table we come to know that 304% Of number always ends with 0, 1, 45,689 unit digit, Square of an lumber car 3,7 ts unit place, i Never have 2, woe Mace Short umes Bat ge wopoursmahboob.wordpress,6QM,, On observing squares ot numbers between 21 to. 29 we get following pattern Qt = 4[4y 22 = alag 23° = §|29| 24° = 576 25° = 625 Note: Hora last two digits are common. Shortcuts for calculation of squares of numbers between 41 to 50.” Ex. Find square of 48? 23.04 08 Square of -2 is equal to 04 so we have written 04 in last two places. Square of 5 is 25, from which we added -2 and get 23. So square of 48 is 2304. 4a? =? 44 is -6 number away from 50 ‘Square of -6 is equal to 36 so we have written 36 in last two places. Square of 5 is 25, from which we added -6 and get 19, So square of 44 is 1936. A=? 42 is -8 number away frorn 50 2 8 + + =25 ae 1764 ‘Square of -B is equal to 64 s0 we ie written 64 in last two places. Square : ig 25, trom which we added -8 and get 17. So square of 42 is 1764 9 Exd 41229 Sol: 41 is -9number away from 50 ‘Square ot -9 is equal to 81 so we have wrltien 81 in last two places. Square of 5 | 25, from which we added -9 and get 16. So square of 41 is 1681 Similarly, we can find squares of other square of number between 50 & 60 Ext §39?=7 Sol: 53's +3 number away trom 50 0 58225 t-—s a ‘Square of 3 is equal to 09 so we have written 09 in last two places. Square of S i825, rom which we added 3 and get 28 So square of $3 is 2609. Ex.2 567=? Sol.: 56 is +6 number away from 50 0 +6 + + 225 Le rs Square of 6 is equsl to 36 90 we have written 36 in last two places. Square of 5 is 25, trom which we added 6 and get 31. So square of 56 is 3138 Ex3 58°=? Sol: 58 is +8 number away trom 50 i + a2 a xe g4 so we Ne ‘Square of 8 is equal to 64° ki ae written 64 in last two places. Suave S —_—_- MADE EASY Scanned by CamScanner oo reasoning 8 Ap yoursmahboob.wordpress.com is 25, rem which we added 8 and get 33. 20 | * Numbor System So square of $818 3364 4 SP =? ‘Sol. 59 is +9 number away trom 50 sa 2 + + St= 25 £-7h 381 ‘Square of 9 is equel to 81 so we Nave ‘written 81 in last two places. Square of 5 is 25, from which we adcied Send get34. ‘So square of 59 is 3581 Similarly, we can find square of other numbers between 50 and 60 speedily. OBSERVATIONS Squore of two digitrumber having in unit places can be calculated very easily nb here n may be any number between 1 10 9. x2 "write 05 in tiest wil two place and * 20 | * Reasoning & Aptiags a Scanned by CamScanner (057 = [nein + 19125 (1gF= 2 We (1X1 + 1))25 = 205, Ba? (257 = [2 x(2+ 1))25 385 4827 [4x4 + 1]28 = 2005 627 (6x(6 + 125 = 4205 as=7 (8x(6 + 11225 = 7205 a=? (x08 + 1)125= 9005 — Mane. similar, we can find Square ofa, also. ne Some Multiplication Technioy, Muttipication usually tekes longe, action and subtraction. While solving.» related to ratio and proportion and pera, we have to multiply Numbers in ang ar. frequently. Here, we have some tricks, 4a make the task of multiplication very erp. Case-! When both numbers are greater than Ext 106 x103 =? Sol: 106 is + 6 number away trom ips, 103 is +3 number away trom 109 106 a6 wo Te $0 106 x 109 will be equal to 10918 (Here either we are adding 1068 30% are adding 103 & 6, in both cass ws get 108.) Ex2 104x105 =7 Sol: 10444 we % 10320 = 10920 Ans x3 108x111 =? Sol: 108 +8 WS 119 8 = 11988 Ans. Beir _aay woN Teast oS = 12705 ef ee Multiplication of 21 and 5 wil 0" woe MADE Mutt we 39 Muto ica wee Zin get 12420 me Sy @ ex yoursm ° carry which will be added to we get 12705 as answer, ExS 118x118=2 (105 +21) and finally M8 stg 118: 418 st A He 13934 Multiplication of 18 and 18 will be 324, we will write 24 in first two place and 3 will become carry which will be added to (118 + 18) and finally we Qet 13924 as answer, Ex6 117x177 =2 NT a7 177 17 136 2/ao Multiplication of 17 and 17 will be 289, we will write 89 in first two place and 2 will become carry which will be added to (117 + 18) and finally we get 13689 as answer. Ex.7 115x108 =? 15415 103 * 408 1241120 Multiplication of 15 and 08 will be 120, we will write 20 in first two place and 1 will become carry which will be added to (108 + 15) and finally we get 12420 as answer. Ex8 125x108=? Sol: 125, 175 00 00 128 78 = 12675 Ans. Exg 111x107=? 1 +1 x 47. 77 41877 Ans. S.CROME Syatem | 21 Ex.10 113 x112=9 Sol: 113 +13 112 126 Multiplication of 13 and 12 will be 152, we will ‘write 82 in first two place and { will become. carry which will be added to (112 + 12) and finally we Get 12652 as answer Case-II When both numbers are less than 100, Ext 92x93 =2 Sol: 92is-8 number away from 100, and 93 is -7 number away from 100. = 8556 Ans. Ex2 68x95=7 Sol: 88is—12 number away from 100, and 98 is -5 number away from 100, thus 8 te 5 5 3 = 8360 Ans. Note: Here we are adding 95 and -12 or 88 and-6. x3 98x87 =? Sol: 9% =E 9 8 x 3 = 8091 Ans. Ex4 89x88=7 Sol: 89, 86 ~ x12 eee Reasoning & Aptitude Scanned by CamScanner a2 | teunwer vr Al 1132 by multiol”ing Here we gel have to 2 end wo gat Note: and 12, where tiscarry. We with 68 and 11 07 89 and ~1 dosired result as 7832 xs 97 x94=? Sol: 97 3 weet nae = 9118 Ans. Ex.6 85x98 =? Sol: 88 2-15 wes 3 wo = 8330 ANS. a om Here we get 324 by multiplying -18 and -18, where 3 is carry. We have to add 3 with 62 and ~18 we get dasired resutt as 6734, Case-Itl When one number is greater than 100 a another number is less than 100. = Ext 105 x93=7 Sol: 105 is +5 number away trom and 83 ig - number away from 100, Bie7 1s 8 aX 5 98 00 9600-35, =e 3765 (©) 13,2 10. Scanned by CamScanner “yoursmahboob. wore . The sum of two numbers is 15 2% SSrCOMN and tinalfesult willbe 9765”, ex2 it Ht x ga =12 99 oo 132 9900 182 9768 Nota: Hero we havoto usta cy, 9900 and final result will be 9763, oooo Cj Solved Examples . The last digit of the number obtained » multiplying the numbers 81 x82 x83 x8tr. 85 x86 x87 x88 x89 will be @o (b) 2 7 (2 Ans: (a) Last digit will be multiplication of 1x2x3 x4 x5 x6 x7 x8 x9 Since 5 and 2 are given here, tte” ‘multiplication will result in zero as lest 49 The sum of the digits of a two-digit u™ is 10, while when the digits are reversed." Number decreases by 54. Find the' number. (e) 2 (b) 19 (©) 37 (9) 46 Ans: (a) Fi Going through options we get 82- 23* a Geometric mean is 20% lower "27 arithmetic mean, Find the numoes (@) 11,4 (b) 12.3 woe uns t Ans ay. ‘@ ©) tc yoursmahb6ob.wordpresSEORS AN ‘MADE EASY = Number System 2 Ans: (b) Ans: Going rough options ony 12 ae satsty ‘ee web bye cond Se B shouldbe oven ame 1243_ bo 16 +B shoud boom : De awsibie by g GM=Vi2x3=6 which is 20% less than 75. 1 A381 is divisible by 11, find the value of the smallest natural number A? @s ©) 6 @7 os Ans. (c) ‘A381 is divisible by 11 if and only if (A + 8) = (3+ 1) ie. (A+ 4) is divisible by 11 So, A = 7 Satislies the condition 5. It 381A is divisible by 9, find the value of ‘smallest natural number A? fs ©) 6 7 oe Ans: (b) 381 A is divisible by 9 So 3+8+1+A=12+4 is divisible by 9 SoA6 6. Find the LOM of 6/2, 8/9, 11/14. (a) 280 (0) 360 © 2 (0) None of these Ans: (d) LCM of numerators_ LOM of fraction =F CF of Denominetors Here, 5/2, 8/9, 11/14, 80 LOMof (6, HOF of (2,0, 14 7. A number 156 is divisible by 6. Which of ‘these will be true about the positive integer 8? (a) Bwill be even (b) Bwill be odd (c) Bwill be divsible by § (d) Both aandc LOM: So only 6's the value for B (@) 24 OB © a (©) None ot these Ans: (a) 100 No. ot Zeros = 122 , 100 , 100 se es (Wrole numbor of) = 20 +4 = 24 9. If 146! is divisible by 5°. then tind me ‘maximum value of n (a) 34 o) 35 ©) 6 oa Ans: (b) 1a AE inom highest value of 5 will be 146 | 146 146 Bote hegre 48+ 1205 10. Find the number of divisors of 1420, @ 4 ©) 15 © 2 (@ 2 Ans: (d) 1420 = 142 x10 = 71 x2x2 x5 = 2 x5! x71" No. of diviger = (241) (1+1) (141) = 12 Note: Number of divisors is nothing but the umber of factors of 1420. 11. Find ne HOF and LOM ofthe polynomials (2-508) ana (= =73+10) (@) (+2) (e-2Nlx-9Nx-8) (bo) (x-2).(e-2)(x-3) ©) (o-hls-2le-Bl=-9) (¢) None of these —— reasoning pitice | MADE EASY Scanned by CamScanner 7 yoursmahboob.wordpress.éqin 24 | * Number System Ans: (2) x2 5x46 =(x-3)(x-2) a -Tx4+10= (-5)(x-2) So, HCF = (x-2) LCM = (x-2) (r-3)e-8) Directions for questions 12-14: Given wo different prime numbers P and Q. Find the umber of divisors of the following 12. Px (a2 we) 4 me os Ans: (b) PxQ=PtxQ? => Number of divisor = (1+ 1) (14 )=4 13, P2xQ (a) 2 (4 os as Ans: (c) PQ Number of divisor = (2+1) (141) =4. 14, P38 xQ? 2 4 6 @e ‘Ans: (d) Pex? Number of divisors = (3 +1) (241) = 12 15.A mikman has three different qualities of milk. 403 gallons of tst quelity, 485 gallons of 2nd quaity and 496 gallons of Grd qual. Find the least possible number of botties of equal size in which different mikcof diferent qualities can be filed without mixing? oi (b) 45 ©) 6 Ou Ans: (d) tis given that mik of Ist quaity 408 gallons Wind quality: 465 gallons Ic quality: 496 gallons toast number of botles of equal sing possible, when we have ots haying, srrargost se, Largest sie page found by finding HCF of 403, 465 4 49" 403 = 13 X31 465 = 15x31 496 = 16 x31 HCF = 31 Total numbers of gallon required = 13+ 15 + 16 = 44 gallons, 16, What is the greatest number of 4 qi when divided by any of the num 12, 17 leaves a remainder of 17 (b) 9793 (d) 9487 ts es § LOMof6, 9, 12, 17 = 612 greatest number of 4 digit divisible by 5:9 is 9792, to get remainder 1 number sigs be 9792+1 17. Which of the following is not a pee: square? (@) 100868 (0) 3, 26, 137 (c) 945723 (Gd) All of these Ans: (d) ‘Square of number never ends up with2,3 7.8 18. Which of the following can never be int= ending of a perfect square? @6 (o) 00 (c) 000 @ 5s Ans: (c) 19. The LOMof 5, 8, 12, 20 will notbe aut? @ 3 (0) 9 ©s8 (as Ans: (b) LOMoF5, 8, 12, 20 will not be aruline®? 20. The LOM of (16 =x) and (+x - 818 (9) ~ ayx4.3y4-22) (b) 414 ~ 204.3) 24 | eReseenegeagiide ere NEN Scanned by CamScanner ade! —— Ny MADE EASY ue, (d) Mere Ans: (a) 16-7 a) (Pree Lom wit ( 21. SCD ot 2 (a) x42 fe) e-2 ‘Ans. (b) Mazts (¥ +4-8) Ged =(r- 22. Decompos Such that t (a) x, x, ‘Ans: (a) Wxty= 28. Which of» divisible by (a) 32188 () 16208 Ans: (¢) Arumoet¢ ands Ony 19624 24. For anumo ve (2) Divisio (0) Duis (or uroceay VOUrsmahboob.wordpress.com oS Numb or Systom | 25 Scat ) (one of these ties hee ‘Arnumber to be divisible by 88 it should be tehdiaipes Givsibe by Band 11 because Band 11 aro Be eTUMberS whose malipication gives (¥ +x-6)=(c+3)(2-2) 25. Find the GCD of the polynomials e+ 3? LOM will (16—22)(2? +2 (2-2) + 1P and (r+ P(e ied cane era 21. GCD of 2-4 andx2+r-6 is (b) (+3) 6-2) (re trea) (@) x42 (b) r-2 (©) (+3) (41)? (©) 2-2 (d) 242 (8) None of these Ans: (b) Ans: (c) 2-4 = (x-2)(c+2) GOD of (x+8)(x-2)(x-+1)’ ang (x +2-8)=(x+3)(r-2) (+9 (<+3)(044) wil be (x43) (e497 26. Find the LOM of (x + 8) (6x24 5x ~ 4) and GOD =(r-2) (22 +724 3)(e+9) 22. Decompose the number 20 into two terms a ee ee 4 hen Such that their product is the greatest. © (42- ee 5 s 14 (@) x, =x,= 10 (0) x, =5,x,= 15 (© (2e- Needitmca: (©) x,=8,2,= 12 — (d) None of these ‘Ans: (b) Ans: (a) (x43)(6x? + 52-4) =(+3)(2r-1)(0x+4) (2x? +7243)(243) = (2r-+1)(x+3)(2+3) LCM =(2x+1)(2x-1)(x+ 3) (3x +4) constant then xy will be maximum 23. Which of the following can be a number = (422 —)fx+9) (area) divisible by 24? (@) 43215604 (b) 2561284 27. The product of three consecutive natural (c) 1362480 (d) None of these numbers, the first of which is an even number, Ans: (c) is always divisiole by ‘Anumoer divisible by 24 will be divisible by f@) 12 (b) 24 gand8 6 (d) Allot these Only 1362480 satisfies the divisibilit criteria Ans: (d) socio at ee consecutive number wil be n(n + 24, For a number to be divssiole by 89, itsnould neal Tie even number then (n-+2) el re also be an even number and one of trern will (a) Divisible by 22 and 6 be divisible by 3. Hence number is always (b) Divisible by 11 and 6 divisible by 12. For exemple if n = 2 then (¢) Divisible by 11 and thrice by 2 23.4=24ifn=4thend 5.6= 120 (6) Botn band c © Reasoning & Aptitude | 28 MADE EASY Scanned by CamScanner yoursmahboob.wordpress.com Macy Number Syste * woMaxesy oes 30 Wer gts ous ve parr Dae or Seve + awsoi cy 2 3. Frome surter ance berg reves: @ suspe ys wi De exacts Swebie by 17.22 Sect @ us nw st 2) Nove sree ‘Ans. (b) . UM? 2 Baas Some cesrec-rser s a . S Te reais unser ess arch sdvstie Dy 48 G0 an 2 0 = a0 © 70 sore 378 As (a) “e-eaea~ sera: ce Te 2? Sete ee gow == rahe WM FSS Sarge SAO Sere ages Sage ese Scanned by CamScanner yoursmahboob.wordpress.com MADE EASY oe fe) r012 (o) 1037 (<) 1090 (8) None of Ans. (a) weno The number 's divisible by 11 and can be Written in form (LCM) +4. LOM of 8. 9, 12 1418 504 ‘So the number may be 508 & 1012 but 508 iS not Givisible by 11 50 the desired number is 1012, 6. Find the greatest number of tour digit which when divided by 15, 20, 28 leaves in eacn case a remainder 2? (a) 9662 (©) 907 (c) 1080 (@) None of nese Ans. (a) The required number will be (n"LCM) +2 LCM of 15, 20, 28 is 420 So greatest number divisible by 420 is 9660, (Four digit number) So required number will be 9662 7. Find the two numbers of 3 digits each of whose GCF is 310 and LCM is 1860 (@) 620,930 (©) 240,540 (©) 720, 1030 (2) None of these Ans. (a) Going by options we can determine that ony option (a) satisfies the given condition, 8. TReLCMof wonumber is 12times their HOF The sum of HCF and LCM is 403. if one number is 93 find the other. (@) 134 (0) 124 ©) (0) None of these ‘Ans. (b) Itis given that LOM=12 times HOF 10. LOM +HCF = 403 So, 13 xHCF = 403, => HOF = 31 LOM = 372 also we know mat HOF xLOM =Number(f) xNumoer(2) 3 91K372=99xN2 + NQ= 124 13 1110 of my income on nouse on conveyance 12ot iigren’s education 9, Ihave to sper’ rent, 1/10 0fremainder further remainder on ct MADE EASY Scanned by CamScanner 10. * Number System 27 aller wnich | have Rs. 648 lett over wnat my income? (a) Rs. 1200 (©) Pe. 1400 (c) Fs. 1700 (0) Nore ot trese Ans. (2) Ore alternate netnog Let supsose | navex ruses Ater spending ctitconnause rent inave Sr % Fo Now out of 2 score Toot On conveyance soreranderw ice Aya oe 0°” t00* "400 1 Further | spent 2 er | spent 3 7 Of Nis property, to me younger remainder, the rest to he widow Fr the Snare of ne sors fine widow gets Rs 3600 (@) Rs. 1200, 1900 (0) As. 6000, 2000 (©) e.7500, 1000 (6) Nore ct rese Ans. (d) Let sucpose man is havingx Re as propery ia peti Be Coen So Ss stemanng ITN oo ee yoursmahboob.wordpress.com The average of a number is a measure of central tendency of a sat of Numbers, In other words, it 'S an estimate of where the center point of a se! of number lies. Average = Sumo! set of N number N This also means Average x N = sum of set of numbers. Illustration: Let suppose the score of Sachin Tendulkar in last § matches are 90, 160, 10, 70, 45 then average score will be Ave 90+150+10+70445 _ 365 _ > % 5 5 Concept of Weighted Average When we nave two or more groups whose individual averages are known, then to find the combined average of the all elements of all the grouos we use weighted average. Thus, if we have K groups with Averages Ay, Ag... Ax and having 1,.7hy- Tk elements then the weighted average 98 gven by formula: A= Tye Mg Mgt vee HM lustration: Let Rahuls average score in 5 testo! maths is 75, 4 test of physics IS 80 and 3 test of MADE EASY Scanned by CamScanner ‘chemistry 15 90 the fing the overall average mars obtained dy him in these subjacts Here we have to calculate weignted average which can be cakulaled by A, = 15x8+80x4+90%3 _ 965 5+443 72 = BOA approx ‘Sometimes weighted average is used tofind merit order of students in competition examinations Like for examples iin a Pre Engineering Test (PET) Sum of scores obtained by two students are equal then their weighted average is calculated to find order of ranking Mlustration: RAHUL & Ravi Scored following marks in PET exam. Teupea Rana Ran] Subect [Rahul [Ravi] Maths [90 [80 | Chemsty [60 [80 | In this case total scores are equal for Ranui ard Rav. To find ranking weights can be gen in following manner. Matns 3, Physics 2. and Chemistry 1 in this case weignted average can be calculated as, Ay(Rahul) = 5 ‘© Reasoning & Aptitude 90 | * Average Ba A gerne 8 Ado) = z 73.33 6 Hore waighted average of Ranul is more than ‘weighted average of Revi, ec Rahul will be oreferred in renking. Some Important Results i 1. Average of frst ten natural Number is 142494010 _ 85 gg 0 10 2. Average ol frst ten whole Number O42 +34 Orta be wae 10 10 3. The average of first ten even number is 244464 70 4. The average of first 10 odd number is 14345¢. 19 _100 10 07" ‘Average of frst ten prime number is 24945474115 134174 19423429 a | 6, The average ct first ten composite number cs 44848494 104124164 18416418 SAB HB 10s 10412 414+ 18416418 ‘0 ~ yoursmahboob.wordpress.com ~ MADE ‘a, Sum of nnatural numbers gett 243tat oot aio 2 9, Sum of squares of first n natural nun, Ga t+ 24 Pt? {aera} 6 40, Sum of cubes of first n natural number; S84 B+ TF Bact fe} 2 44, Sum of first n odd natural numbers, S H143454..... (2-1) s=rt 12, Sum of first n even natural numbers S=2+446+..2n aa(ns tenten Cy] Solved Example 1. The average age of 24 students ani" Princisalls 15 years. When the pnncipalsa 's excluded, the average age decrease 1 year. What is the age of principal? j @ 38 0) 0 | joie (6) Data nacon’ { 7. The average of ston odd pk © \ . Helos nie Avetage age of 24 students { 945474119 13417419429. sumot 1 TT + 29 + 29, = SUof ages of 24 | a0 9428431 x students: = 14 = SUM Of ages of 24 students 24 30 | © Reasoning & Aptiugs ~ ~—— ing & Aptitude Pe _ wanes oe Scanned by CamScanner wan 2 » © © —yoursmahboob.wordpress.G@iy ++ SUM Of ages of 24 students = 2414 336 years Let tne Age of principal be x year Then, average age of 24 students and he erncivel = SUT of ages of 26 students & principal SEE 24 Stunts & principal aS =15= 25 = r= 25x 15-396 = 39 years 2. The average weight of S men A, 8 and Cis 844g, Arctner man D joins the groun anc ne average now oecomes 80 kg. IF ancther man E. whose weigrt is 3 kg more than that of D. eplaces A nen the average weight of 8.6.0 ang E vecomes 78 kg. The wegnta A s (@) 70Kg. (©) 72kg. (2) 794g. (9) 78ig Ans. (c) ‘Sum of weignt of A.B and C x64 = 252 «g Sum of weights of A.B, C and 0 = 4x80 =320 1g weignt of D = 920-252 = 68 kg 2. weight of E = 68 +3=71 kg Now, sum of weignt of A, 8, C, 0 anc = 320 +71 =291kg sum of weight of B.C, Oand E 24x78 = 31245 weignt of A = 991 - 312 = 79g 3. Three years ago, the average age of A. 8 and C was 27 years and trat of B anc C. 5 years ago was 20 years. AS prasert age Present sum of ages of A,B, ard © #814943 = 90 years Sur of ages of 8 ere C. 5 years age = 20 x2 = 40 years "resent sum cf ages of @ ang C = 40 +245 =50 years AS Present age = 50-50 = 49 years P Vijay Tenculkar nas certain average for 9 innings. In the tenth inning, he scree 100 runs thereby increasing nis average sy Bruns. His rew average aa (b) 26 ©) 2B @ 2 Ans. (c) Let the cla average and new average oy 2 and y respectvely Then, y=r+8 ) 9:+100 and y= 10 , From () anc i) we get 1Ote+8) <9 100 = 22 y= 18228 The average of the frst five mutioes ct 7s @a (e) 21 2 aa ‘Ans. (b) Fist fve Tuttioles of seven are 7.1421, 28,35 7. average of first ive muttoies of 7 70+2+3 2] The average age of 2 fariy of 6 memcersis 22 years. If ne age of the youngest member ne T years. wnat was the average age cf he family at che birt of the youngest Nemcer? i 5 we pass (©) 35 years 2 2 (c) 40years 0 Sees Ars (b) ee Ws (0) ages AB ana.0, ere years alee ee oor x3 = 61 years Reasoning BActtce 5 Scanned by CamScanner yoursmahboob.wordpress.com x & ee firs ot age ct Haney eben, yah AN SAS BN Te Dyas axeaaye ave of fy her at the Bvt" Ao yest enter vol = Shee fe avenge sue a 8 deren ert carmmittor se ieanent ty 2 veces nines nacre aged RS Yeas ant aS years are sudstitted bY NC ware Heatwave age oF ENEMA ws wei as we wa Ans. (a) Let the average age of 8 persone in the derveas Lotte sum ov ages MOWONET HEY VES. Prensa ol agesc! Boesonseciuding Me meraged Sand 4s years oS GSH) Wee 8 oon Agen cate we Sur GaSe y aren 2 aeerage age of We women en & The speee of he ran © going trom Nagour camrened « Woievtesharareree ne beck hom Alsnabed ohn, tecewede ithe Prd penwage mensoorgie ance EUmey @ 1 ws © i a ans (a) Let the asians Denwees Alanatad and Nagou be whe The average sceed = ht too * 150 32 ® Reasoning & apntuce: " MADE LAs, EX10ON 18) 20 yn “9 Note Avelave speed Total distance taroigs Total tine tan he aver age WEIDMLOEACIISS OF. shader ag 4 ag HE the WEIGME OF Me teacher bg inclatod, the average rises by SDN Why us the wag of the teacher? (dong ©) 50540 &) adhO (a) SS kg Ans (d) ‘Sum of weight of 28 students = ADA29 = 1160 AQ Lot the wought of the teacher De 14g. ther Dex woro5e VS SS ve dOSKIO= 1160 = 1215-1180 The average of 3 numbers is 17 and thatt the hrst too 16 Find the thd number @ ss @) 16 ow @ 9 Ans. (a) Sum of Snumber «3x17 =St ‘Sum of the frst two members= 2.4 16 = 2 ou The thir mamper = 51-32 = 19 Treage of Mth and Poo sin tne rae 2 & After S years, the ratte of their ages wil become 8:8 Find the average of Mew Ages after 10 years @ 12 ws or wa Ans. (a) Letine prasertage of Mita) ane Maa be. and & years eseectvely aes 8 ater S years, « Ges 8 FL HATRED = 2r- 10 MADE EASY ee Scanned by CamScanner Mann 6450 mur wopeirsmahboob.wordpress.com 2 Hence Present ages are + anc 3 years After ten years, their ages will be 11 and 19 years ‘Average of their ageg 144 =!89 aan 12.Find the average of ine first 97 natural numbers, @ «4 ©) 48 ‘Ans. (4) ‘Sum of fist n natural numbers (0) 37 (a) 49 alae) 2 ‘Sum of the first 97 natural members Their ==. er average = == 49 13. Find the average of all prime numbers between 30 and 50. (@) 298 (©) 388 () 37.8 (@) 418 Ans. (a) The prime number betwoen 30 and 50 are 31,37, 41, 43, 47 2. Theiraverage 814974414 43447 5 14, Tne average ot 5 consecutive number is 1. the next two numbers are also included. the average will 98 15, 16. "7 * Average 4 Given natitltre2e essere 5 snexe2 The average alter including the next two numbers aELEAOx+ Qe re Se redersSere6 — 7 _ Fx+3=n-243 =net Hence tne average increases by 1 ‘The average of SO numoer ic 36. It two rumbers, namely, 45 and 55 ate discarded the average of the remaining numbers ig fa) 265, a7 (ce) 378 (8) 375 ‘Ans. (d) Sum of 80 numbers 38 x50 = 1900 ‘Sum of remaining numbers = 1900 ~ (454 =. Average marks obtained by a student in 3 papers is $2 and in the fourth paper ne Obtains 60 marks, Find nis new average, (a) 54 (@) 82 (©) 55 (535 Ans. (a) 3x52+60 Trenew average = °*5% The average weight of 5 men is decreased by kg when one a them weighing 150 kg is replaced by another person Find the weight the new person? (@) increase by 1 (@) remainthe same te) 165 ifs Cj romeoris @oemene Ans. (a) weight of 5 men be x kg era Let the average Laine Re exmocttio nies 2 before replacement of one ers07 x4 2.x4¢35,ande * Reasoning & Aptitude | 33 MADE EASY es EE Scanned by CamScanner «| +eursmahboob.wordpress.com's, Also Let me weignt of new person bey kg 18. The average age of a group of men is nereased by 5 yeas when a person aged 18 yea’s 's replaced by a new person of age 38 years How many men are there in the group? @3 {o) 4 es me Ans. (0) Let mere den men in the group and ine average age of the grous De x years belore replacement 18 +38 7 > ee Sent 20 ones Tren +5 19. The average age of the inaian cricket team Saying te Nagour test is 30. The average ageo!S of Me Diayersis 27 and that of arotner Setof 5 players totaly dfferert from the trst fre. $ 29 If 18 the captain who was not included in etre of these two groups, then thd the age of tne captain, @) 75 (©) % () 50 (2) Carnot pe setermned Ans. (c) Let the age of tre cantan be x years Trane Bis Haiesvin x5 3 3802195404 145 + x= 550-280 = 50 years 20. A bus goes to Ranchi from Patna at the rate 1 604m per now. Another bus loaves Ranchi ‘or Patna at the same Ie as the toy, tne rate of 7047 Ber NOU Find the soeed for the JOUMeYS Of the ty comoined ifit SANOWN that the distance ranchi to Patna s 420 kilometers (a) 6481570" (0) 64. Skemp (c) 63.823«729— (B) C4. 82 amp Ans. (a) ‘The average soeed 2x420__ 2x60%70 _ 840 “20 420 =~ 730 13 60 * 70 = 64615 «mon 21. Out of trree numbers, the frst $ twee re second and tee tres te the The aver of the three numbers is BB. The smaice number is (@) 72 (0) 36 (o) 42 (©) 48 Ans. (d) Let the first number ben. then the other two noon rumbers willbe 2 ang 2 Swilbe > ana 3 non net Agan, ag = ""3°3 Tre smateot number = 2 = 144. 4g 22. The average wegntot 6 persons is creased Dy 2S kg wren one a nem whose weght is 50 kg is reo.aced by a new man, then the ‘weight of the new man is 34° | © Reasoning & Aptitude Scanned by CamScanner (@) @5%g (©) 754g (©) 76Kg @ 6g MADE EASY 24. a cl A a a MADE EASY Ans. (a) Let the average w. be xkg before reslacement Lot the wt. of new men be y kg, Then, x+2,5 = ¢¥=504y > 6x+15=6x-50+y = y=65 23. The average age of three boys is ‘5 years. Ii their ages are in the ratio :§ : 7, the age of the youngest boy is (a) 21 years (b) 18 years (c) 15 years (@) 9years Ans. (d) Let the ages be Sr , Sx and 7x years Then, 15 = S#+5x+7x 3 = 15=5x, x23 the age of youngest boy = 3x3 = 9 years 24. If a, b, c, d and e are tive consecutive odd numbers, then their average is fa) S(a+b) (b) (abodeyS (c) Sla+b+ec+dedte) (@) None of these Ans. (d) Average = alate (are (ar6)+(ar6) 5 a+20 5 25, The average of first five multiples of 3 is @3 9 (©) 12 @6 Ans. (b) sara 14243444 5) average= A sass ot 90 studen's avorage weight of ac * be kg. It, however. the weight ofthe teacher MADE EASY Scanned by CamScanner yoursmahboob.wordpress.com * Average | 36 27. 28. 's included, the average become 41 kg. The Weight of the teacher is fa) 31kg (0) 62kg (6) 71kg (&) 70K Ans(c) Lot the wt. of teacher be x kg Then, 41=40%30+ x at = e541 x31-1200 = 1271-1200 = 71g ‘The average weigh! ol aschocl of 40teachers is 80 kg. If, however, the we ght of the princple be included, the average decreases by 1 4g, What is the weight ofthe principle? (a) 109kg ©) 29kg (©) 39kg (@) None ofthese Ans. (0) Let the weight of princiole be x kg 40x80+x Then, 79 = © 4 =9 x= 79x41-3200 += 3239-3200 = 39 kg The average tomperature of 1%, 24 3'¢ December was 24.4°C. The average temperature of the first two days was 24°C. The ternperature on the 370! December was {@) 200 (ys (o) 2520 (6) None of these Ans. (c) Let the temp. on 3° of December be x C x24+x Then 244 = PRESS 2520 ‘The average of 20 results is 30 and that of {90 mote results is 20. For all the results taxon togetner, the average i= (a 5 (0) 0 @ 2 (9) 24 ~ Reasoning & Aptitude | 9S yoursmahboob.wordpress.com, 36 | © Average Ans. (d) The required average 2043049020 - 50 20, The average tempereture on Monday, Tuescay ‘and Wednesday was 41°C and on Tuesday, Wednesday and Thursday twas 40°C. Iton ‘Thursday t was exactly 39°C, tnencn Monday, the temperature was (@) azo (0) 4 (6) ae (@) we Ans. (2) ‘The sumo temp on Tuescay ano Wecnesciay =3x40-39= 81°C ‘The cum of temp. on Monday, Tuesday and Wednesday = 3x41 = 123 ‘Then temp, on Monday wit! ba 123-31 = 420 1200 24 Average Ds 1. The daily earings of a taxi driver during Week are As. 60, Rs. 65, Rs. 70, Rs. 62.50, AAs. 63, Rs 73and Rs. 68 Whetishisavorage ally earring for the week? (@) Rs 7450 (0) 564.50 (©) Resaso (@) Rs6a50 2. Theaveragect 1Onumbersis 7. What wil be the new average it sach cf the numbess is ‘multiplied by 8? @) 8 (e) 82 © & (a) 55 3. There are 35 students in a hostel Hf the number of students increased by 7, the expenses o! the mose were increased 96 |» Reasoning & Aptitude Scanned by CamScanner MADE Fay by AS. 42 per day while the avers, expenditure per head diminished ty ne > Find the orignal expenditure ofthe mess (a) As. 480 (b) Fs 440 (©) 38 520 (3) Ps. 420 ‘An aeroplane travels distances 2500 jen 1200 km and 500 km at the rate of 500 key 400 krury and 250 kmmUN resDectivay hg average speed al the aeroplane is (a) 420k) (0) 410 kent (2) 40Skmhr (@) 575 kent The average weight of 24 students of secon ‘Aoof & class is 5B kg whereas the average weight of 25 sludents of section 6 of the same class is 60.5 kg. Find the average Weight of all ne 50 students othe clzss @ s74kg (0) 595 «g (0) 58.9k9 (0) 59.7 4g) 3. The average ago of 5 members is 21 years ltheage oftne yaungest member ke 5 years, find the average age ofthe family at netrn of the youngost member (@) 24 years (©) 25 yoare (©) 20years (©) 28 years Tho average monthiy salary of a stalf of 9 persons is Hs. 2450. One member of the Stafl whose monthly salary is Rs. 2650 8 transterted. Find the average salary of fe emaining & persons of the staft (a) Fe. 2405 (0) Fs 2625 (o) Fs. 3025 (a) Fs. 2825 |. The average ot fve consscutive even numbers staring wit 6 © 6 7 fc) 8 @ 7s . S years age the average age of a family of ‘Smembers was 17 years. With me birth of a ‘new Daby the average romansthe same ever today Find the age ot ne baby, MADE EASY Mec tase 11 Aparsren 185 ona 2 Whaisne Nenadneve wo os 12 Tre ane erween *! teetwoon se: secon ax (0) 8 18. Anaroer 5.7.14are andy $28 a3 os 14h mearesa averagea Wee oD ne bs % piv ty pe ny at 4G yew fe BME pene Ny f¥p YM NNR Np IRN 64 OOS WA pe Go Ag sangre % 8% ig thy te $9 Aemcrer : NY ee. mete ST DR ORY HOLE Spy Ot BING BANGED cr gt Toe ee nee HE OE x ae ty 4 t FL AOD SURES te VOL LENSER GAT G9 te MORAL OSE TAY EN Or Ye WIENER 6 4% “iy the Me br ip brie ES Om enraged hasan, t8 warn SE why Ot Ke bed Oe “4 ed 0s t, te 6 GT Het! OL WILLY TH VOLPE OO LLL NM BALL AASB LE MD BS NGS My LE he BOL WLS ROS” Pada sh Sart BPE EOE APE OD seemesviy WS Ye erage "5 “nneraeg open wr Wt PF Y, pata te Wh pA QAM AOE oe EDO 4 OPH ir Se Faas 1 Hef al ale Wi EE Scanned by CamScanner + wv % % _yoursmahboob.wordpress.com thet 4! "8 MOV MAS 88 60 GH 5 MAB 081 90m PUB AHA “4 LOWE MENDON agin tn 4, Btw i weve MONE 8 Bb rrce OR BANG MEET) 808 § 1 0 Seeprag og op 10 Ms om tanswos 4 OS wip 60 eee ee LB TOR ea 8 WU GIE ORE Ke ering De Renee. OOH ret 44% 644% ett @ 474 ABD OID MI Boy Pan At Mes YR GLE IT HG a8 LA Hy ON 8 BEN ner” er, ene othe erat own om amr 8 Tate sa BI EE 8 Ip DE SIA BUR, OT ETNA DS Bete aaa, we 8 ED AOR ER OES . a MS OAL Tie ERO AAI TAGE, OM LRA Sg PE BE LIMEES DN Ee Pe ca a nett, 6 PNA COTW oo wee ee 3) 6 2 23. The average cay wages 7A Bans As -20 PS eans As 40 ore man C oer day ard Aeams owe Fara CaaS say Te wages oA De Cay S$ a Psa 2) Rs *20 i) Rs 160 3) Ps 0 24. Wor ara average sosed Ff em ty a “eacTes ts Gest Tater anime Fi gpes ern an average specc Of HS wr ts late Dy *$mrues. The weal preys a) Oe fo) ak ic) 70K () = = 295 59.3kg 50 6. Ans. (c) Total age of 5 members = 21 x5 = 105 year Total age of 4 members al the birth of the younger member, thats, § years ago =105 - (5 x5) = 80 years . Before the birth of the youngest member, the family consisted of only 4 members. MADE EASY Scanned by CamScanner 10. =2e8 5 5 Ans. (d) Present age of 5 members, = (6 x17 +3 x5) years. = 100 years. Present age of 5 members and a baby = 17 x6 = 102 years. .1. Age ofthe baby = (102 100)yrs = 2years. Ans. (a) ‘Sum of first nine numbers + sum of last nine number = 10.5 x 9+11.4 x9 = 21.9x9= 197.1 Hence, the middle number =197.1-17 «10.9 = 197.1 - 185.3 = 11.8. - Ans. (b) ‘Average score before 17th innings = 85-3 x17 =34 s, Average score after 17th innings 34+ 3 = 97. '* Reasoning & Aptitude |= 39 yoursmahboob.wordpress.com 40 Average 12. Ans. (a) Wis given thet A.B =2: Sand B.C=5 8 Combined ratioct A.B Cwilbe 10:15 24 Since +B+C= a8 therefore A = 20 8=30 8 Note: Please reter Ratio & Proportion to ‘ind out Comoined ratioof three numbers. 19. Ans. (c) StT+idey, Average of5,7, 1dang yx 2t 71682 S474144 Wis ghen tat x= 80% of S274 144y 80 26+y 00" @ wo va 2bty 0 iso 2+ 226 iso “aa “ From (iyand ((i), we get Seay = 26 iy aty 252 (iw) Solving {it} & (iv) we gel y = 33 14 Ans. (c) Let the taintali on Wednesday be x om so that on the otter 6 days, the ota! i also x Since average rata for he week = 3 cm 3 44=3x7 or,x=105 0m 15. Ans. (0) Lat the average age of 8 men be x years =, Sum of the ages of Bon « & years Now, according to tne condition of the Queston, average age ol 6 men +2 women) (© + 2) years Sam ofthe ages ol (6 men + 2 women) = Br 42) = Br + 16 yoare 40 | * Reasoning & Aptiude Scanned by CamScanner MADE EAs, Hence, itis clear thal on replacing two mg, by two women, sum o thei ages incre: by 16 years ‘Therefore, sum of the ages of two women = (20+ 24) + 16 = 60 years .. Average age of two women 6 = 5 = 0 years 2 70% 16. Ans. (D) Total distance covered Av OO Tosh time taken 100 +100 +100 700, 100,700 ~ 98-3 kmite 30 * 40 * 50 17. Ans. (c) Weoght of D = (804-84 x3) kg = 68 kg Weight of E = (68 + 3) kg = 71 kg (B+C +0 +£)8 Weight = (79 x4}kg = 316 kg “(B+CYs weight = (916 468 + 71)]kg = 17TKg Hence, Ae weight = (84 x3}-177}kg = 75iq 18. Ans. (d) {90 by option 19. Ans. (a) Correct average 8036-73457 _ 1764 Ee aokg Lethe number ot candidates who passed «1 Then, 39 xx +15 2(120- x) 2120 x36 * 24x= 4200-1800 2400 or, 0 28 8 aq 77700 21. Ans. (a) Total dectease = (20 x2) ments es MADE EASY 22 An 24. An 25.6 le hiscecae d “ 4x8 x5 = 160 = Man. day hour = 160 x 2 (tivtce as carter) 2x20 xh = 1602 h In & moture of 40 litres, tne ratio of milk and water is 4:1. How much water must be added to this muxture so that the ratio of mik and water becomes 2:3 fa) 20 litres (©) 32ites (© 40inres (8) Dives Ans. (c) Lot rater is x 4xcx mike water 92:8 = 4x. x(since toia! 40 ters) 32.2 axe 40 litre It three numbers are in the ratio of 1 2.3 and hal! the sum is 18, thentheratioct squares of the numbers is, @) 6-12.13 (oy 12-4 (©) 36: 144 324 (a) None of these ‘Ans.(c) Let numbers are 1: 2x: Ar also tis given that 1 Zr x4 3x) = 18 30 3x) Or =96.x=6 6:12:18 ratio ol squares 36 : 144 324: oralso 1:4 9 . The ratio between two numbers is 3. 4 and their LCM is 180. The tirst number is (@) 60 (0) 45 (©) 15 (2) @ Ans. (>) Let number are Sxand 4x Se xr = HOF x 180 Clearly HCF wil oe x 12 =x x180.4= 15 rumbers willbe 45.60 (@) 6 (o) 7% © 8 (a9 MADE EASY Scanned by CamScanner © Reasoning & Aptitude | 47 ae yoursmahboob.wordpress.com 4a | * Ratio & Proportion Note: if there are twa numbers N, & Nj then Ny Ny = HCF of (N,N) xLOM of (Ny, Np) 7. The incomes of A and B are in the ratio 3 : 2 and thelt expenditures ere in the ratioS 3. If ach saves RS. 1000, then, A's income is (@) Fs. 3000 (b) As, 4000 (©) Fis. 6000 (a) Rs. 9000 Ans.(c) Let incomes ave Sx 2r expenditures are Sy. 3y then e—5y = 1000 (i) also 2e-3y = 1000 il) from (i) ana (i) we get &x— 10y = 2000 > ‘A’ income is 6000 8. ihe ratio ot snes of angles of a miangles is 1514 VB. then the ratio of square of its Greatest side to sum of the squares of other two sides is fa) 3:4 (b) 201 fo) 121 (9) Can'tbe determined Ans. (c) SIMA: 8inB:sinC= 1:1. JB sinA:sing 21.1 angle are equal So itis a ight angied tangle, 4 as 33) c B the AB? (AC + Cay MADE 9, Divide Fs. 680 among A, B anes A gets 29 of wnat B gets and 8 ge, what C gets. Now the snare of C 2 ia) Rs. 480 (©) Rs. 300 (c) Rs. 420 (d) None of Ans. (a) 660 =A+B+C 2 % A= 5B andB= 2¢, ap; 2 = 5B.C=4 A 3 C=438 20 .a+c8 17, B. B= 120,C = 480 10. The students in three batches ai Made Eas, are in the ratio 2: 3: §. If 20 studerts xe increased in each batch, the ratio change 4:5: 7, The total number of students in tne \itee batches before the increase wore (a) 10 (©) 90 (©) 100 (2) 150 Ans. (c) Let students are 2e ar: Sx now ‘According to the given condition 2r +20: 38420: 5x4 20=4:5-7 x=10 20:50 50=2:3: Sand atte- agang 20 40:50:70 11. The speeds of three cars are in the -ato 2:3 4, The rato between tne times taxer by these cars to travel ine same distance (@) 203-4 ©) 4.32 () 4:3:6 M643 Ans. (d) ‘Speeds are in the ratio 2 rato of tme taken wil be 5.4.3 i212 1g =F 4:3 48 | © Reasoning & Aptitude Scanned by CamScanner An * wae Ae & Nene ” yoursmahboob. wordpress.com YS Ate ay Siemertat * ad donnrmnans, a tacten heen Aw (0) both as via ay ithe onynathacton WN fa ate (ov t9 are four nutes was @) an velvety ny Ans (0) Voy = We ve f we? 3 1B. W the tats OF the ages of Maya and Chhaya wera {86 Sat present and ittenn yeaa hoo pow ALS EW dee Ped tho tao wal got changoct tod thee trad ONIN y= 278 satishes the given condition Meao’s present age: (a) Mayors 0) yeu 19. The aittorance Dehween MMO positive NuMDERE 1) W years Ryewe vs 10 and the ratiobenwaen them is 3 Find Ans (b) the product of the two numbers Lot thoi ages age G80 tN, A a3 (175, trat (ey 27S (d) 125, . Ans. (a) Bras 9 yoy = 10 w pets a rye5 3 wy 48H HTD 9 aS #1 roy Bt roy according to (i) and (ti) Bt -3t = 10 Biss Ohlins 17 1ERe 888 divited among eee Pan that onic anil ant eater bow got Product oktwo ndmibers'=-S76 Fospectively Tran now many gts are there” 14. The present ratio of ages of A and Bs wt oe 45 18 years ago, this ratio was W168 (c) We Find the sum total of their present ages Ans. (0) (a) 90 yours. (b) 105 yours Lot the number of girls and boys ate candy {c) 1Oyears (d) 80 years thon O20 1 Ofye Be 0 Ans. (a) 2 Lot ages are 4x Sy then itis given that vote a 4-80 trom (ihn (a) ve get sx-18 16 Ody eee 6 Gay ~ 288 = S5e~ 198 vey I , F Qe = 90,0610 90 Oty att ee BB sages are 40 and B0 94m = SO 11.91 aan ies tenant v gin tho ratio 1:9: 4: Fadd up moron eras raw We oft SMA, Tawm y numbers : i ° oo fa gum of 105, Find the valun ot Ue at et biggest number a re ® ms Ma es ue a oa oe (ec) # twawoning A Apunaie | MADE EASY Scanned by CamScanner _ kQuasmahboob. wordpress.com Ratio & Proportion Ans. (6 wus ‘Ans. (a) Let number are 5x 4nd Bx then it 1.2.3 6:89 5x49 _ 8 be 2347 42 ae Hi Bananas are inratio 6x Bx x tvs te ntine . Itis given that a aes {ist monkey get 102 bananas 22. ix varies as y, andy = 7 when x 18 ey 'o 19, Amistute contain milan wate nth rio. wheny. 21 as 5.1. On adding 5 tts of water, the ratio of (0) % (o) 4 fen ik to water becomes § 2. The quantity of (o) 72 id) 8 25 he mmik in tre rigtura 6 An (b) in (@) t6ltres (©) 25tires «sweetly proportional to v wt (c) 325 lines () 22.75 i006 $0 xeky “ Ans. (b) 18 = 7 ) Let milk and water are Sx a 36 be Now, 225 5 10e= 6 +25 " x52 te br = 25, Now = Beat asa cubes malate e610 23, vatios yay as B and C, and A = 6 we: 20, Vijay has coins o the cenomination ot Re, 1 B=3.C=2tndAwhonB= 50-7 ¥ 800 and 25 p in the rave of 1210.7. The (ay175 (0) 56 . total worth of the coins he nas in Rs. 75. Find © 70 (a) 105 % the number of 25 p coins that Vijay has ‘Ans. (b) a (a) 48 to) 72 Ler A & (BC) tran () © (3) None these. © =KI32) ‘Ans. (d) . 8, = tthan Coins are 121 10x: 7x A = (BC) 7 =1x(5 x7) = 36 ftis ghven mat 12r+ 10x 242 e 2 75 at 24. Wr vanes as y directly, and as 2 inversely Mie t Bet Te 76 and r= 14, wheny = 10. find 2 when x = 49, 4 yas 4 Tr 4x75, x04 (a) 14710 a (0) 10 21. iHtwonumbers are intneratioot§ Bangg (6) 196 be added to each, the ratio becomes (8) Cannatoe derermines 2 11. Now find the lower number Ans. (a) (a) 6 (0) 19 , () 15 (2) None of rose eaty ie wa MADE LASY — ner rrernenrn ST TTT __ Scanned by CamScanner yoursmahboob.wordpress.com soos Bastin te Proportion Jf Paces eon wz ag aa Usp eB ee TTT Scanned by CamScanner yoursmahboob.wordpress.com-. 52 | # Ratio & Proportion thal betwoen the second and third be 7 9 than tne thrd numbar @) 94 () €4 © 8 (6) None of these 12, The ratiobetneen twonumber is 2.3. eac? umbor is increased by 4, the ratio becomes 07 Thenumber aro (6) 8,16 (6) 24,92 (o) 16.24 {) None of those 13. The ralio of present agos of Suresh and Mahesh is 7 5 ttaltor 8years heir agos wil bbe in the ratio of 4 : 3, the present age of Mahesh (@) S2 years (©) Soars (b) 36 yoars (@) None of hese 14, Tivo numbers are inthe ratioo! 57 It 25 be subtracted from each, they are in the ratio of 95:53, Findthe dtterence ol the wonumbers (a) 46 (©) 82 (©) 24 (2) None of these 18. The value of k that must be acded to 7. 16, 43, 79.s0 that they ate in oroportion. is (a7 5 (9 (8) Noneot hese 16. In@ number that must be added to each of We numbers 8, 21, 13 and 31 to make the ‘ato of first ive numbers equsl to the ratio of last Wwonumber is @s 7 oo (2) None of these 17. The incomes of A and B are in ne ratio3 2 and their expenditures in the ratio § 3. it ‘each saves Rs. 1000, AS income is (a) Rs $000 (©) Rs 6000 (©) Rs 8000 (61 None of nese 18. Amature contains alcohol and water in the fate of 12. § On acing 14 ives of water * Reasoning & Aptitude: Scanned by CamScanner MADE ty tne ratio of alcohO! 10 WalEr DBC ¢ The quantity of alcohol in tn8 meture (@) 1Bites (©) 42 ites {e) 25ites (2) None ol mesg Solutions ‘Ans. (b) Letxbe the fourth proportional, hen se 60 _ 30 60:48:98: or = indi 0x28 _ 9, is 0 5 ‘Ans. (a) = Letxbe the third proportional, then 8 he K Bsc eee 3. Ans. (a) a Let consequent be x 115 re ae ++ Coneequent = 70, 4 Ans. (b) . Let be the mean proportional. Then, “ 025 >: 004 re ozs of O28 a) ae o e001 © r=01 nes 5. Ans. (by Let first and second number be 3x8 FESOBCIIVoly It 'S given that “ Bar = 420 = xs60 SoWet rember ae = 189 umber 4x = 249 wer es maw cmon 6. Ans. (b) Let the shares of Amit, Sum be 3x, 4x and Sx respectively Its given that Br + dr + Sx= 660 12e = 680 5 =55 So Se = 275 7. Ans. (b) Let the price of Scooter and Television be 3x & 2x respectively Wis given that 3r - 2x = 600 ject anc Puneet Television's price 2x = 1200 8. Ans. (b) A:B =7:5 “ "1 i) Bis common in eq. (i) & (il) ‘To equate the ratios of 8 wo will multiply eq, (i) by Band eq, (ii) oyS Then A:B = 63:45 B:C = 45:55 AB: C = 63:45:55 9. Ans. (c) A:B oli) Ail) Bis commen in eq, (') & Gi) To equate ratios of B in ea. (i) and (ii) we will multiply eq. (i) by 5 & ea. (ii) by 3 then A:B = 20:15 il) B:C= 15:12 (w) Hence A:B:C = 20:15:12 (b) TOA og a4:5 0 B:c=5:6 ‘i Bis. common among them 8 ae menu aay. Sana 4 money Sani re eg 6. since Ani hat niu : MADE EASY Scanned by CamScanner “—yeursmahbo: Ob. wordpressshi0aM F's. 280, the amount of money Mi arjutas 280 = SP x8= Rs. 420 11. Ans, (a) A:B 22:3 BC=r:9 wo = AB:C=14:21-27 A+B+C = 124then C=54 (i) 12. Ans. (b) Let the two numbers be 2x & 3x thon 244 8 Baa 77 > 14x428 = 151420 = So numbers are 16 & 24 13. Ans. (c) Let ages of Suresh & Mahesh be 7r & Sx ERB A on ach Br+6 ~ 3 on oNng We get x26 Then present agaot Mahesh e.Sx=0years Then 14. Ans. (c) Let the numbers be Brand 7 Sr-25 _ 35 SEE = & on soins 7r=25 7 59 OSONND Wego, x= 12 Difierence betweentwonurbers Tx Sr = Or = 24 15. Ans. (D) ‘According 19 given conditon Tek 164K: 4o+k: 784% 74k | 43+k =) jesk ~ 794k going by options we can tnd tnat« = 5 '» Reasoning & Aptitud® yoursmahboob.wordpress.¢ MAD} 54 | * Ratio & Proportion 16. Ans. (a) Let xbe the desired rumber tne Bex _ 1348 Bisex ~ Stee going by options we oan easily find that re5 17. Ans. (b) Let income be 3x: 2x and expenditure be &y : 3 then Sr ~5y = 1000 2r~ By = 1000 = x = 2000 = y= 1000 So A's income will be dx = 6000. 18. Ans. (b) Ratio of Alcohol & Water is 12: § Let Alcohol be 12x then Water willbe 5x tax 4 Now Ber id 73 3 38x = 20x + 56 18 = 56 = ¥ So 12r = 42 itors oooo Partnership 1, Aman and Pranial enter into a partnershig investing Rs. 50000 and As. 40000 respectively. They agree to share prolits in the ratio of theit capitals. Find me share of ‘Aman in a profit of Ris, 22500 altar cne yoar {a) Fs. 12500 (0) Rs 9500 (©) Rs. 10500 (A) None of those 54 | * Reasoning & Aptitude Scanned by CamScanner MADE Fag, oe : 2 Amit, Nitin and Ravindra entered in, partnership. Amit invested RS. 169q9 7 8 months. Nitin invested As. 12099 ,” 6 months and Ravindra invested Rs for 12months, Atthe end of a year there was a profitof Fis. 26000. Find the share of Ny, in the profit. (@) Rs. 8000 (0) Rs. 6000 3. Sakshi starts business with Rs. 3500 ang 5 months afte: Divye joins Sakshias her paring, After @ year the profits are divided in the ratiy of 2:3, How much did Divya contriaute? (@) Rs. 7000 (b) Rs. 11000 () Rs. 9000 (a) None of these 4. Arvind began a business with Rs. 550 a0 \was joined afterwards by Naveen with Rs. 330, When dic Naveen join if the profits at the end of the year were divided in the ratio 10.37 (@) Alter months (b) After 8 months (c) Alter 45 months (d) None of these (b) Rs. 7500 (d) None ot these 5. A, B and C invested capitals in the ratio 3:5: 9; the timing of their investments b2ng (nm the ratio 2: 3: 1. In what ratio would ther profits be distributed? (@) 2:5:3 (0) 3:2:5 (©) 7:5:3 (d) None of these 6. A. B and C start a business. If the ratio ¢ thelr periods of investments are 2-3. 6.and their profits are in the ratio of 4 § : 6, ine? the ratio of capitals of A, B and C is (a) 6:8:10 (0) 12: 10:6 (©) 10:12:6 (d) None ct these A. B.C and D enter into partnersnip. A 1 Vad Subscribes 3 ofthe capital. B 7. C Zand O the rest. What is the share of D out of a proht of Rs, 60007 (a) Rs 2000 (©) Rs. 1200 (0) Rs. 1600 (9) Rs. 1300 ——_ MADE EASY ‘om * woecrsryoursmahboob. Wordpress GQM +s 8 A and B started a dusiness with initial ‘vestments in the ratio . 7. latter one year tett profits were in the ratio 1 : 2 and the ericd for A's investment was 7 months, 3 invested the money for (@) 6 months (0) 2% (©) 10 months (4) 4months 9. A.B, Center inte @ partnership with shares in 7.46 theratio 5: 5: Alter 4montns. A increase is share by 50%. If the total profit atthe end of one year be Rs. 21600, then B's share in the profits (@) As 2:00 (6) Rs 2400 (c) Rs. 9600 (d) Rs 4000 10. Bisa slooping porincr anc A working Aputs in Rs. 5000 and B puts in 6900. A received 12.5% of profit for menaging the business and rest is divided in procortion to ther capitals. A's share of profit in a proft of Rs, 880 is (a) Rs. 350 (b) Rs. 400 (c) Rs. 420 (2) As. 460 11. Asiarts business with 2 capital of Hs. 1200. B and C join with some investments aiter 3 and 6 months, respectively, Ita the end of a year, the profit is divided in the ratio 2:3 : 5 respectively, what is B's investment in the business? (e) Rs. 2400 (0) Rs. 1800 {c) Rs. 3600 (a) Rs. 6000 Solutions 1. Ans. (a) = 50000, C,= 40000and P = 22500 Here time period of investment is constant So ratio in which profit wil be divided is Gy: p => 5K: AK = 4 Here, MADE EASY Scanned by CamScanner Now Aman's share. = 322500 = Re. 12500 Ratio in which protit wil be divided i.e P,P, :Pyuill Be Cit, Cat: Coty 16000 x9: 12000 x6 - 8000 «12 = PLPy Ppa 623.4 Nitin's shar 8000, Ans. (c) Let suppose Divya’s contribution is then We have, © x1, = 3500 x 12 = 42000 and Cy xt, =x x7 = 7. Protit tor Sakshi The °° “roti tor Divya or, r= —** = As. 9000, Divye's contribution is is. 9000. ‘Ans. (b) LetNaveenremainin the business ferxrnonts We have, C, x1, = 550 x 12 = 6600 Arvind's share of profit _ Cit Naveen's share of profit C, xt 6600 x3 = = 6 months. 33010 =e Ans. (a) Ratio of capitals of A. Bend C are 3:5 9 Let the capitals of A. B and C be 3x, Sxand Gx, respectively. Ratio of timing of their investments are 2:3:1 Let A, Band C invest their capitals tor 2y, 3y andy months, respectively ‘© Reasoning & Aptitude | 55 ‘ yo tio & Proportion Binks ota profit of B profit of C mont Cpe Oa* oe xy: BE SY | EHD ng: 15.9012 5:3 . a ih profit wi be dvided Le Py: Pz: Pai Cs Ce We have, P,:P):Pg=4°5 andt, ty: =2:3°6 Then, protit ify 6 Py Py. Requited atio = Sits = 4 4.5. ®t 23 ola or, 12: 10:6 Thus, A, B and C invested their capitals in the ratio 12: 10:6. 7. Ans. (d) D's Capitel = Proftrato gf A,B,C, Dis 2:4:4:48 345 8 . 20.18:12:13 Share ct D= 22 Rs. 6000 = Ri a =Rs.1300. 8. Ans. (c) Let investments ot A and 8 r espectivel Sx and 7x and period of B's inves! bey ursmahboob.wordpress.com ~ _ g. Ans. (0) Given ratio = = 108: 40.35 1 Months. tment be y thon, OX*7 1 (ajay 7B AP=10. Qoo00 Scanned by CamScanner Lat the initially INvESt RS. 105, Rs. As. 36, respectively. * Ratio of investments [105 x4 + (150% of 105) x8}. jo, (86 x 12) z 1680 : 480 : 432 = 95: 10:9 erates Rs(21600%12). ae Ans. (4) As share for managing the business = 125% of Rs. 880 = Rs, 110 Remaining profit = Rs. 770. Profit ratio of A and B = 5:6 A's share= Ti of Rs. 770 = Rs. 350. A total profit= Fis. 350 + As.110 oft Ans. (a) Profit ratio of A,B and Cis (1200 « 12): (e x9) x6) =2:9° Taking first and second terms W® ot 1200 x12 :9r= 2:3 = 1200 x12 x3 = Or x2 gq 1200x123 p40 18 LA Thet deriv frene The fact com isat date Int per 9 ge zp yoursmahboob.wordpress.com Percentage YZ The term percent means “for every 100°. It s derived trom trencn word "Cant" winich is 100 for trench The basic utlty of percentage arises from the fact that itis one of the most powerful tool for ‘comparison of numerical data and information. tt \s also one of the simplest tool for comparison of asta In the context of business and economic performance, it is specilically, useful for comparing data such as profits, growth rates, magnitude and s0 on. Calculation Snce percentage represent the value oDtained ‘out of every 100, calculation can be done in following manner Value obtained ee 100 Percentage = “Terai value For example, let in IIT JEE examination Saryay scored 135 mares out of 300 marks, Then Marks obtained By Sanjay in % term is equal to 5 4m percentage is aTrongst tne As discussed ear iter, for example most powertul toot for comparison. let MADE EASY Scanned by CamScanner ‘Shweta scored 120 marks cut of 150 and Ranul scored 360 marks out of 500 then Percentage 120 score of Shweta = 122 = 80% 150 360 Pe =7i ercentage score of Rahul £5 = 72% Thus, it clearly shows that Srweta’s performance is better than Rahul's performance. Other Comparison Let in Annual Examination Megha obtained 200 marks out of 300 and Sangeeta obtained 100 marks out of 300, Now to compare marks Of Megha and Sangeeta two question arise 1. Megha’s marks is how much percentage greater than Sangecta’s marks? 2. Sangeeta's marks is how much percentage less than Megha's marks obtained? Megna 200 Sangeeta 100 Clearly Magna’s marks is 100 greater nar Sangeetas marks. Its means dttorence of marks . 100 _ 199% Sangeeta marks 763 Now Sangeeta’s marks is 100 less than Meghass tt means ~~ Reasoning & Aptitude | 97 oursmahboob.wordpress.com on 2 Percentage to the conclusion tnal ume to the conclu is 100% greater than Sangeeta > ‘5 marks 15 50% less than Meogha’s marks. ‘Some Useful Shortcut Methods 1a) If Ais. % more than that of B then B's less than that of A by linr™ % 100 +x 1(b) It Ais.c% less than that of B then Bis more than that of A by = «100]% 100-x fx 1 Shailendra’s salary is 20% more than that of Surendra, then how much percent \s salary of Surendra less than that of ‘Shailendra? Sok Horex= 20 Required Answer =| * qui Her [iogex*t00}x 0 4 100% = 21 = 16.66% Ex 2.116 income ts 30% lass than that of B's income, then how much percent is B's income more than that of A's income? Sol Here x = 90 Pequred Aeame: =( FE wor [a 100} 30 m (i100) «aon 2 WAls x% oC and B 8 ¥% of C then A-(Zim)na 8 | * Hoasoning & Aptitude Scanned by CamScanner HAG 20% of G Ex. a what percentage | sol Here x = 20, y= 25 20 * 4100 = 5= 100 A aie 3g *100 3a) i the price of 2 Commodity ing. by P%, then the reductic, consumption 50 as NOt 10 increas, expenditure is : = x100}% 300+P 3.(b) I the price of a commodity decies, by P% then the increase , consumption $0 aS Not to decteas, the expenditure is P (seeo*100)* Ex. 3(a).If the price of sugar increase by 25% Find how much percent its consumation be reduced so as not to increase ne expenditure? Sol. Reduction in consumption orp* 100) 25 = 705 %100% = 20% Ex. 3(6) If price of commodity decrease by 2" lind how much percent its consumo be increased so as not to decrease”? expenditure? Sot: Increase in consumption P «100% 100-P wee a o o Mh the “ bx ay Sol. Ex Sol. yoursmahboob.wordpress.com MADE EASY 4 It number is changed (incteaseay decreased) succousivaly by .% and y% inen net% change is given by yi (oe 3%} whic roprosont incroase 0F decrease in value according as the sign in (+)ve oF (-)ve. 1x01 y indicates decrease in percentage then put (-)ve sign before x or y, otherwise (4)ve sign. Fx 4(@) Il salary oF a porson ie increased by 10% and 20% successively then, what ‘8 the changg in his salary? Sol. Hore x= 10. y 4 20 The not % change in the salary 10x20 700 = (10 +204 ) = 32% Ex. 4(b) The price of a commodity first increased by 20% then decreased by 10% then what 's the net change in price of commodity Sol; Here x = 20, y =— 10 then net percent change in price is 20x(~10) - Nw [2-1002C} son Hore sign is (+vo) hence the net is (+ve) increase in price. Ex. 4.(c) If price of a commodity decreased first by 20% and then by 30% then find the net change in price? Sol, Hore x = 20, » = -30 Net % change is [ Ee we 20-304 Tae Since sign is (~]vo tha net change is reduction in price ation of a town (or 8. If the present popul Valuo of an item) be P and the population MADE EASY Scanned by CamScanner * Peicemags 59 (OF value of item) changes in 1% per annum, then (4) Population (oF value of an item) after 1 years (ub) 100, () Population (or value Of an item)n years: ago= (a) 100, Where + is (+ve) or (~)ve according as {the population (or value of item) increase or decreases, §x:5.(@) The population of atown increases 105 ennually. It its present population is 120000, what will it 02 in 2 yeers time? Sol. Here P= 120000, Ex. 5.(b) The population ofa town increase at the fate of 20% annually due to excecsive migration. I present population is 144000 find population two years ago, Sol: Here =P = 144000, r = 20 ‘Population of the town two years ago. Pp =, ("ico} 144000 __ 144000 = 5 = py = 100000 ( a8 2) ¥00 “100 6. Itanumber A is increased by «%, y% and 2% Successively then final vaive of A will be a(: arse a) a Reasoning & aptiude | 59 r F j « \youxsmahbeob.wordprass.com In case if 2 givon val percentage, We will us before that. py 10% Ex. 6. The income of Ramesh neroase 2 find change in his #nco income is 150000 Re. 0, 2 = 30. 0, percent changein income dvishr sla) 110, 120,180 _ps.257400 =180000 «99400 “100 1. To convert traction into percent i to convert any fraction =~ into percent rmuttiply it by 109. 3.3 For ex 1. & = 2100 = 60%, 6 2. To convert a percent into a fraction ‘To convert percent into fraction, dr Sinandcide re nonberby ise rece, S é 100 = 25% 14. Ans. (b) Feduetion in consumption -(aee x00) 1 1 : ee oF 13-16, (woex} 8 15. Ans. (b) Here, x = -50 and y = 50 The net % change in wages xy = at Is ~(vo et} 50x50 100 Since the sign is -ve, he has a loss of 25%. or - 25%, =[(40+50- 16. Ans. (b) The equivalent discount of two successve discounts of 20% and 20%. MADE EASY Scanned by CamScanner EE yoursmahboob.wordpress.com 99 4 20%20 = [20-204 S22 lo or — ( 04 en 36% Given. 36% ~ 35% = Rs. 22, Amount ot the bill = 22 x 100 = Ris, 2200. 17. Ans. (¢) The equivalent discount of two successive discounts of 20% and 10% = ay. (rv+ 35} 20x10) (20-104 700 so 28% Discount on the lst price of radio offered by the frst shopkeeper = 28% 01 1000 = 22 «1000 =As.260 - 100 Also, the equivalent discount of two successive discounts of 15% and 15% - (« ++ at) 15. 1 Discount on the list price of radio offered by the second shopkeeper. 3 1 = 273% of 1000 = 111 27 39% of 1000 = 77% x 1000 = As. 277.50 Ditterence in discounts offered by the two shopkeepers = Rs. 280 - Rs. 277.50 = Rs. 2.50, 15 3 =(-19-15+ a io 0 275% 18. Ans. (a) Since tax xconsumption = revenue Net % change in revenue y - oan | (++) 10x10 =(-19+10 100 x Scanned by CamScanner (note x = 10 and y = 10) = 1% The revenue decreases by 1%, 19. Ans. (a) Since 4 x1adius xradlus = suttace ae, Net % change in area 20. Ans. (b) Since side xside = area Net % change in area ¥ (eevee 30x30 = (20204 0 =60% The areas increased by 69%. 21. Ans. (a) Since side, xside, = area Ertor % in area = (« tyt (Here, x = 10 and y =-20) = 12%, 19, 12% deticit. 22. Ans. (a) Since tax xconsumption = expenaiture Not % change in expenditure ay aoe 20%20 = (20-20-52) fremay--a = 4%, Expenditure decreases by 4% iy ' Bai \yoursmahboob. wordpress.com HAT AT joan AWS Ly Mant g hos Tre yyy ALY AY . oe ate ! Veus any WH RIA ay, oP Saar VANN@ WRU UKDG white Magy Sends ‘) . aout at ds (al) bey! A BW a eo aby hn = POR 100,100 00 Ns gi LG Na ha yaa Sy 6 Waveho 400 stints ayatane 1 Mh 24 Ans (0) OSU AHON Here, Ae ROND,y 5 Wye Mand ex a. 20 An (by Value ot We Hinate WH allay A years Phys kay, RD aft a Pasa (93 9 CENTLY GSR aD =A : ( 10 " wo hill +a) aed eee Fait WWD FW Ve mn) aa} 40 8 = 8201 a te ( wo Won) (' 1) Fats [Aran tS a os BONO 90x a0 Wf 4 Ve D0 40 Reman 28 Ana (ny 100% 100% 100) . Hostatbn = Mic hy Ph Ans (a) Wea “4 $atA be the original strength 20 Aww (o) ‘ y e Now visa al ha wasiabunt Thea a(n wa)! voll ins) ' = 789000 (Givany m- ni) Now se Wye Wathen 10 0 i my WW AD AL (a) K 00 ‘ si mn) m Hid Wise Fats ti 1000) aah) a F000 8 100% 100 100) days Re a6 HO) 100000 inn) ‘ o “ a on hat sehuastesili piaiesing Vee COT) Haw isitiay A Hina) Aye fd MAL APE Scanned by CamScanner , ssepoursmahboob.w \ percentage fi) yractic? & OTT HSE " eopte cage 7 a wes : wows wee ao a) 2D 2 eect apecatrwotithot anemeet gs Ww ape 40% ‘of that number? ww ww 2) 2 (a) 1 3. Wratvatbe 160% of a number whose 200% sar (a) 0 {p) 160 140 (a) 12 4, Wines fitth ofa numbers 40 more than 40% ofthe same number, What is the number? fa) 100 (p) 190 (©) 20 (a) 400 5, IIx is 90% of Y, What percont of X is Y? (@ 1014 (©) 190 © 9 (dit 6. (weotyty%otx)=? (a) x%oty (b) y% of x (c) Mot ry (@) xy% of 3 7. 190% of A = 30% of Band B = x% of A. then x is equal to: {a) 900 (v) 900 (c) 60 (a) 600 8, 40 quintals is what percont of 2 mele tonnes? (a) 20% (o) 2% (©) 200% (a) 180% 9. Astudent has to secure 40% marks to pass. Ho gots 178 marks and fails by 22 marks The maximum marks are: mz) Reasoning & Aptitude Scanned by CamScanner oxdpress.com (a) Wve Mane Jo ore total arrmuunt menived by ant on PURERNET AINE rng ost te@Rsportatend Wwe tn, 10 Ms 19, theo inital aTNOUNE WA \ 2000 (b) He dw (a) Ha Nooo 11 The price of jute has Bear reduced by {tthe reduced price ® RE WOO porque, WA the original Pree: per quintal wars Y (a) Rs 900 (b) Re 610 M (6) Rs. 960 «) 100 : 12, Amit has Some APHIS. Ho gold aoe nano ate. tthe sold 70 apples howmm — & apples did he eat 2 ’ q@ 50 (b) 9 aw @) 8 (a) 42 13, ayo partat the popu inaaviiage sens It 30% of the males are married, Ue porcentage of unmariod emales: in tho tet population ts i (a) 70% (by 40° wo) 2? U % (a) 0% 14, Aschool has only three clases ich conta ‘to wd andt GO.tuctonts rospoctivaly Te Pa of these classes aro 10 joctively The pass percontage oly (b) «) 6° 3 1. 30% of Ay salary is equal to 20% oF, B's salary, IH B's salary fs Ry 2400, wh" As ualary? wanes” = 8s "SEC ee 2 AS ES 39s 260 1S TENA stTesarea oe ney ts ox Fas oe Ent Note > Baca > Nore of rese 37. Asmara 7 Bowgy s 2 ess Tar Bad Te Sts Taree contained ty er Boog Mats arc Drawing iris wares - Oraeirg ce SC wrat ss Tans mr Mars? 2< rs = 2 Cer toe ceterminec Scanned by CamScanner ~——yoursmahbaob.wordpressecom.. 22. ar wareater Mee we 3 cae Mamerates of 100 ars esc a secures ON + rez cae re 4 - Te secore cece ir woe © secure” Te aggregate te cercemage 9 - arc -z Csecse r Trt sce © x ™ < Es x a, 2B. Tec -ur cers are ess Tar ates“ ery 30% anc 27% cess 4 Moe Tac percert s Te secorc -urcer est tar Te fa? 2 oO 7 ™' ns 27 24 pereereae Ag NTC este BapRtraecsete Tscla meester lt tA RR eS 4 Percentagt while 15% taleainbotn te subj. 12500 ‘andidatos appeared at re examination’ trary pessoa inthe subject bu cen BO (a) (b) 1175 (© 21 (@) None of these ins ina collage are inthe Febo * deka het boys.and 25% of the girls are adults, the percentage of students wo ars not adults is. (a) 5% (¢) 78% 29. The price ot sugar is increased by 20%. As & reawt, a family decreases its consumption by 25%. The expenditure of the family on sugar will be decreased by: (@) 10% (0) 5% ©) 14% (a) 15% (0) 675% (©) 825% 190. A building worth Rs. 189, 100 is constructed on tand worth Rs, 72,900. After how many years will the value of both be the same It land appreciates at 10% p.a. and building depreciates at 10% p.a.? (@) 25 ) 2 © 18 @3 31. A reduction of 21% in the price of wheat enables person to buy 10.5 kg more for fs. 100. What is the reduced price per ka? (a) Rs. 2 (b) As. 2.25 (@) Rs. 2.30 (@) Rs. 2.50 32. Sataries of A, B and C are inthe ratio 1: 2:3. ‘Salary of B andC together is Rs. 6000. By what Percent is salary of C more than thet of A? (a) 300 (to) 600 ©) 100 (6) 20 33. Thelength ofa rectangles increased by 80%. By what percent would the width have to be decreased to maintain the same area? t@) 374% (0) 60% (©) 75% (@) None of these 74 | © Reasoning & Aptitude “yoursmahboob.wordpress.com 4. 35. ‘ Mig For a sphere ot radius 100M, then, value of surface 2162 IS WHat perc,” numerical value cf its volume? (©) 265% (a) 45% Actickt team won 40% of the tot ‘of matches It played during a year jy, ‘50% of the matches played and 20 ma. were drawn, the total number of mg. played by the team during the year vas (@ 20 (o) 100 () 0 @ 40 Solutions Ans. (0) 25 5 =x = 0605. 25% of 25% = => 95 Ans. (d) Let the number be x then Syde@arets ‘Then, x = 15 x30 = 450 40 =o x 450 = 160, Now, 40% of 450 = +55 Ans. (d) Let the number ba x then 200% of x= 140. 200 Than, 555 %# = 140 oF = 70. Now, 160% of 70= 182 70 = 112 100 Ans. (c) Let the number be x, then 3 Br 7 40% of x= 40 Scanned by CamScanner ware o woe koursmahboob.wordpress.com ——e—es 3 ~ Bm ao 3 2 gr g7240 o ze O ¥=(40x5) = 200 5. Ans. (a) wn}-(Becofeers 9 11, Ans. (a) 6. Ans. (c) BOR otr=80= Z r= 80 100 =80x Zee = rary 12. Ans. (a) ‘Suppose he ate 1 andes: Then apples sod = 140% of x 2300 8. Ans. (c) Note that 1 metric tome = 10 quittals: 2 metric tonnes = 20 queta’s Required percentage =(Sxro}s 20 20 9. Ans. (b) 40% of x= 178 + 22 100). so0. of Soren os-(~5) _ Scanned by CamScanner 7s_| yaussmahboob.wordpress.com Unmarr (S2\ ned females: ¢ a)" 18 «Required percentage Seid 7 =|—x- = 27=%. (Sixtieo)naard 14. Ans. (a) Number of passed candidates 10 sou 20 50.412 = (29 x40+ 20 x50422 (ios 01 Ba s0-755x80) = (4 +10 + 6) =20 20 a Passed percentage = 735607760)" 20 1 -(Zxseo)n=193% 16. Ans. (c) sas rho a= ¥% of b = GAs TGP D x @)a-(2)5 y vi 2% of v= (a ozs y, xz a * (Sale-(5)* ote 17. Ans. (4) Let B +M+De=zx. Then, B = (25% of x-20) wis sft is given that D = 50 np * _204M+50=x o Me It . 4 ace $0, marks in Maths cannot be de 18. Ans. (¢) - Let total quantity of original milk cr Milk atte trst operation = = 80% of 1000 = 800 gm. Milk after second operation = 80% of 800 = 640 gm, Milk after third operation = 80% of 640 = 512. gm 24 -, Strength of final mixture = 51.2% 19. Ans. (a) Let A's salary = x, Then, B's = (2000-r) 8% of A =15% Of B, 2 § 15 78 (2000- = 1500 400" 700000") & * 20. Ans. (c) Sugar xan} =120 gms, Water =180 gms Lat x gr sugar may be added Then, 20% 100 = 50 300+x => 240+ 2 =300+x = x =60 2 21. Ans. (b) Let original price = 100. New price = 82 Increase on 80 = 20 Increase % -(Bx00} = 25% 22. Ans. (b) 804704 x= 72 200 or = 8% 76 | * Reasoning & Aptitude Scanned by CamScanner as WADE EASY + Porcentage 17 22 Ans (a) Lat mea numee’bex Then 3) 5] s 24. Ars. (c) Hy 518 % pnp 30! ana co ob Be Caracal 0 a Faled n 1 supect ory =(27S-375) = 500 a 4 Falea en 2° suspect ony = (1050-375) = 875 BaF x980 = 400. C= 52400520 Passed 7 2% ony + Passac m tsi ony i = (675 + S00} = 1175 mes, Xone 28. Ans (c) ie S.ccose Doys = 3x ang g°S = 2 : Pecenages 0 =( 22x 120) 50% 500 Not aowts =| x3) 25. Ans. (bd) Let ne given traction De x'y 50 29. hos (a) Ltorgra crerroter =o ansb rors poe = BS TCCAnt Segre exoeratse = Ae (190x100 = Re. 19000 New excencte «Rs (*20878) = 48 00 Devease r ewsrrone + Reasoning BApioe 77 WADE EASY ae cw nen eed anneal Scanned by CamScanner roo yoursmahboob.wordpress.com 78 Percentage -(t0 x10) 6= 10% 70000 30. Ans. (d) 40Y _ 10 J reee(1+ 22) = 1a5100%{1 a 44’ ,( 10) _ 133100 _ 1931 (2) (2 = 72900 ~ 729 rane * () -(3) =n=3. 31. Ans. (a) Let original price = Rs. x/kg. Reduced price 10000 — 7900 = 10.5 x 78x ig) xe 2 “705x798 . Reduced price 79 2100) rT oa) snezit 32. Ans. (d) Let A =x, B = cand C = &r, ‘Then, 2c + 3x = 6000 =x =1200. A= 1200 and C = 3600 Required excess «(ae 100 = 200% 7200 33. Ans. (a) Let length = / and breadth = b. Let the required decrease in breacin 460, (100-z) —| x sb Then, Ta5!% 499% =9 160(100-x) = 100x100 30000 OF 100 =" =e 125 1 ws x={100-128) 278 34. Ans. (c) Surface area 4x? = 3 = =x Volume axvou When R =10, we have By-(3 x10} of V=30% of 35. Ans. (a) 40% of x + 50% ofx + 2 =x 40 50 ae SS _ = 7007 7007 * 20 xOrx= 200 Scanned by CamScanner Profit and loss are part and parcel of every commercial transaction. In fact, the entire economy and concept of capitalizm is based on the so called ‘profit and loss’, Business transactions have now-a-days become common feature of life. When a person deals in Purchase and sale of any item, he either gains or loses some amount generally. The aim of entire business is to earn profit. The commonly used term in dealing with questions involving sales and purchase are: Cost Price The cost price of an article is the price at which anarticle has been purchased. It is abbreviated asC.P. Note: Cost price can also be written as CP only. Selling Price The selling price of an article is the price atwhich an article has been sold - It is abbreviated as SP. Note: Selling price can also be written as SP only. Profit or Gain han the If the selling price of an article is more t MADE EASY Scanned by CamScanner rsmahboob.wordpress.com Profit and Loss Cost price, then there is a gain oF profit Thus, Profit or Gain = SP.-C.P Loss If the cost price of an article is greater tnan the selling price, then the seller suffers a loss. Thus, Loss = C.P.-S.P. Profit and loss are always calculated with the respect to the cost price of the item Profit Profit% = rofit oF 100 Loss Loss% = x 100 oss% = 25x Example: By seling an article at 500 As. Moran incurs SORS gain then find cost price of that article CP. =SP-Gain CP. = 500-50 = As. 450 Example Ramesh purchased a radioset at Rs. 1500 anc sold it at Rs. 1200. Find loss incurred by rim? Loss= CP. - SP = 1500 - 1200 = 300. Also in this case we can caloulate © Reasoning & Aptitude 79 — yoursmahboob.wordpress.com \... 80 Profit & Loss BO |e Protea toss 300 = 22 100 = 20% Lose = 32 «100 = 20 Thus he incurred 20% loss. Basic Formulae 41. When SP and Gain% are Given than 100 =(—100__) esp. = (oosem)* 2. When the C.P. and Gain % are given then 100 +Gair% ae 100 cp, 3. When CP. and loss% are given then 5. If the cost price (C.) of m articles is equal to selling price of n article, then % gain or loss ~[PSt}100 ttm >n, itis % gain and if m x110 = Rs, 100 W SP. of rto books «Ro #10) = Rs. 121, Profit % = 21% Ans. (d) CP of 25kg = As. (15 x 14.50 + 10° = As, 347.60 SP. of 25 kg = AS. 25 x15) = Gain = Rs. (375 - 347.50) = Ans. (b) 19. 100 é =As|— x96 |= As. 80% Mean price (3 a x96) Rs. By the rule of alligation: Cot Ghee Cr ot eo" 20 Mean Price 80 Required ratio = 30 : 20=5 yoursmahboob. wordpress.com 15. Ans. (ay ———- Gains (2 B09") rag, 16. Ans. (ay 8 ©. Pol 3 tatees = fo. S. Pot 3 totes = 150% of he 4 3 For Rs. 5, tottees soid «3 For Re 1, tottoes soid = (oxZ)a2 17. Ans. (b) Let S.P. of 45 lemons be Rs. x 80: 40= 120: x07 $= 10 22, x o xs 6 For Rs. 60, lemons sold = 45 For Rs. 24, 40x120__ 80 lemons sold -(Sx=)-1 Ans. (b) 110 x = (1004 P): 2x OF ) x p=120%. Ans. (b) ‘ee 120% of 128, 120 4 21500 A= 180099335" 700 2 Ax[rssoxg} 08 Scanned by CamScanner 21. 110 10042 19942220 2B x ot * Profit. 1 Pettey ws or 3p tae, P(X") seco Ans. (¢) Let CP of a mixe TV. be Rs. y, Then, 2c +y = 7000 ands + 2y = 9600 Multtiolying equation (i) by 2 ane sub Uracting equation () rom &, we ger 3) = 19600 -7000=12600 ory = 4200 C Polal.V.=As 4200 ‘Ans. (b) ToS. = As 24000 CP oforse 8 DE RS. x and that of 3 0 =#{ Baran) aero C.Rotace =fe( Berane) 9000 J CP = Rs. 25000. Pies cena Ae Ans. (c) Lerorgna’S Poe RS = Ans. | in ws heawrsmalhaas. wordpress. Gom a 5/6 ~ $x i tee actual weight is 80% of 1000 gm o x9 96kg = 800 gm 27. Ans. (a) Let C.P. of each gm be Re. 1 40 Then, C. P. of this packet = Rs. 800 OP. 75 xSP. to. SP. S. P. of this packet = 110% of C.P, of 1 kg. = (S100) ot cr =(HBxs000) = = Rs.1100 SP. = 250% of CP. Gan=(329«100}%1=37 5% 28. Ans. (a) 800 Let C.P. be Rs. r 25. Ans. (b) (105% of x) ~ (95% of x) = 6.72 Let the total value be Rs. x of 10% of x = 6.72 Value ot 2rd= 2%, value otftd= 2 eerey 29. Ans. (c) Tas. 2 (108% ot 2+ 96% oz) Lot original C.P. be Rs. x then, (2102 , 98x) _ 908% 300 300)” “300 308: 308x-300x _ 300 7% 00 apg = 400 now 5p = 10, 198 _ 208% lew SP. = 325% 5 = so aaRE 100" 20 ~ 200° - = 15000 2tx 208 Oe _ AX = tor r= 200 26. Ans. (c) 20 ‘Suppose, the quantity sold at a joss be x ko 30. Ans. (b) and let CP. per kg be Re. 1 Let original price per dozen be Rs.« Total CP. = Rs. 24 New price per dozen= 25% = 3 Total SP. = 120% of (24 —x) + 95% of x. 100 4 _8 49x _ 576-24r+19r Now, 96x-4.-% + ag Rt 20 OM OSE 576 -5x oF (128 - 96) = 4x or x = 8. 20 31. Ans. (b) 578-5 410% of 24 Total CP. = Rs. (12 x4 +16 x2) =" * S.P.of 6 dozen organes 94 | © Reasoning & Aptitude mane Scanned by CamScanner at 32. 38. 37. =Rs 96 ape teenineaetom S.P per dozen = Rs. 15 Ans. (b) Let CP. be Rs. x 900 -x =2 (x- 450) ar CP. = Rs. 600, gain required = 25% sp. =a (755x000) Ans. (b) Let C.P. be Rs. x. Ax-75) Ans. (b) Let marked price be Rs. 100, Then, S. P. = 90% of 80% of 70% of 100 (3 80 100 * 100 “100 Bo x:00)=504 Single discount = (100 -50.4)% = 49.6% Ans. (d) Cash price = 95% of 90% of 80% of Rs, 300. 20_,, 82 g00} =" 205.20 1 Ans. (¢) Let CP be As. 100: THEN = As. 130. sp =(109- -2)m of R510 7 375.490) 12" ar5 *\400 21875 . 21% prot =21-879 = "7000 Scanned by CamScanner 1800 ==600, = (96 =x) = Sx= 2461-82 marked price 38, 40. a — 29 discount = 500). (96% of 500 + 496 064% of 6 =Re| (ia ROO ry sa) =Rs. 192.80 Difference = Rs (200- 192.80) = Rs.7.20. ‘Ans. (b) CP. =Rs. 159. Gai 20%, sp.=( x4 ; @ 153 |=Fes. 183.60 Ley, the matked price be Rs. x ae 183.60 > 12830410 9g 5 Ane. (0) Let, the list price be Rs. x 8, y= 242100 ideale) Required SP, = 70% of R530 = Rs. 24 Ans. (9) Let the C.P. be As. 100. Then, SP. = Rs. 112 Let the printed price be RS. * 90 etigea caret? hen, 90% of = 112 99 (ag (ve #90 1120 . CP) (printed price) = 100 a) 900: 1120 =45 56 i i ‘ p.of each article D®| Re.1. ee Gof 20" = Rs. 27 ere ——Fpaasonnge Aone | ae Scanned by CamScanner Simple Interest Itis calculated on the basi: borr S of ao: owed for the entire period ata particu ne of interest particular rate ‘The amount bor! i j period of conowng. fete pincpal for thserir gi = PX RXT 100 Where P is Principal R is % Rate of interest Tis the Time duration ‘Amount A is sum of Principal P ard SI so, A=P+SI Note: Princigal P is also refered as Principal Amount. In such condition also, Amount Is different from Principal Amount ‘Amount = Principal + Interest incurred Interest may be simple interest ‘ior compound interest Cl. Compound Interest The interest of the principal for interest. R cis [te7o3] =F MADE EASY Scanned by CamScanner =e i®) 72226 B89 ~ Weare fy & Ans (c) =) NONE ot Mase Rs 725247 5 Tap, TRE Rese 3. What wi, De The smoie Mprest 733 7 Ita PR bet amar ta he asa to Faun E-Ms DAs saoer 18. Neste ene names soe ec sores BRS) ARB = Rss 2s 34. Ag) nomaws As 7500 Pov wo TOT fenders Te nevs rmsrest st hea De ETT ty TE er oe Ne aD De aT ne ane, Te ts mere Deus tor Mes" e S085 Ne DOT BLE TEE 2 3s 0X Seasons BA Scanned by CamScanner yoursmahboob.wordpress.com 18. It the difterence between the simple interest and compound interest on some principal amount at 20% per annum for 3 years in Rs. 48, then the principle amount must be (a) Rs. 550 (b) Rs. 500 (c) Rs, 375 (d) Rs. 400 Ans. (c) Here P = 2, R = 20%, T = 3 year Ditference Rs. 48 PRT a} 48= sat lintel, Pf aps ez) 20 20x39 ee eee On solving we get P = 376 19, Raju lent Rs. 400 to Ajay for 2 years, and Rs. 100 to Manoj for 4 years and received together from both Rs. 60 as interest. Find the rate of interest, if simple interest is being calculated. (@) 5% (b) 6% (©) 8% (0) 9% Ans. (a) PRT: , PePoTe _ 100 100 2) 400x2%R 100% 4%F _ 5p 100 100 12R = 60,R=5% 20. In what time will Rs. 8000 amount to 40,000 at 4% per annum? (Simple interest. being reckoned) (@) 100years (b) SO years (© t0years —(¢)_ 160 years Ans. (a) P= 8000, A = 40000 R= 4%, SI = 32000, T=? Six 100 T RKP 32000 x 100 8000%4 = 100 years 21. Asum of money becomes 4 trne interest in 10 years, What ig 1 interest? (@) 10% (©) 30% Ans. (c) Let sum be x then A = 4r, SI = x PRT (b) 20% (d) 40% 100 So, R = 30% 22. A sum of money doubles itself in 5 yees how many years will it become four fols interest is compounded)? @) 15 (©) 10 (©) 2 (9) 12 Ans. (b) Let sum =x of R = ate ae= sft] = 4=[y > 2am8 Li = 3° T= 10Y08. 23. Asum of money placed at camioou"s doubles itselt in 3 years. In now 72" will it amount to 8 times itself? 102 | © Reasoning & Aptitude Scanned by CamScanner @) 7 Fa eee i « sxesvounsyrahboob.wordpresstom Ans. (a) (9) 7 years Let sum =x Then, are {1 +B 106 R = |e [ el =e wali) & = = 8 = [aval > B= 28 T > tea S T = 9¥08 24, Divide Rs, 6000 into two parts so that simple interest on the tirst part tor 2 yeers at 6% p.a, may be equal to the simple interest on the second part for 3 years at 8% p.a. (a) Rs. 4000, Rs. 2000 (b) Rs. 5000, Rs. 1000 (©) Rs. 3000, Rs. 3000 (d) None of these Ans. (@) Let one part be x As. then x2x6 _ (6000-2) x88 100 ‘100 42x = 144000- 24x 6x = 144000 x = 4000 714 of itself in simple interest ey becomes tain rate of 25. A sum of mon MADE EASY Scanned by CamScanner T= 8 years, TRE 25% 26 Seriay Borrowed Rs, 900 at 4% pa and fs ae at 8% pa. for the same duration Hes id to pay Rs, 364 in all as interesi, What fe time period in years? (@) 5 years (©) 3years (©) 2years © Ayear at ) 4years PRIM PaRoTe 100 * 100 ~ oe 4 1100% ST. 700 100 367 +55 = 384 T= 4 Yeas 27. If a certain sum of money becomes double at simple interest in 12 yeers, what would be the rete of interest per annum? 1 at b) 10 @ 8 e) @ % Ans, (a) Here P =x, A= 2rs0SI =a then, wax = "400 @ 4 100 gi 21 aay @ Reg "3 1s, 600 amounts to Rs. 720 in jest. Wnatwillitamount jeased by 2%? 28, A sum of Fi = onnprs inurest a LUNpYMes 93 ——=“““yoursmahboob. wordpress.com Sixt pu S1%100 _ 120x100 _ oy, PxT — 600x4 at 7% Rate = B00x7 x4 100 A= 600 + 168 = 768 s! = 168 Simple Interest Practice Exercise: | 4. Atthe rate of 6% p.a. simple interest, a sum ‘of Rs, 2500 will earn how much interest by the end of 5 years? (@) Rs. 150 (b) Rs. 700 (c) Rs. 750 (a) Rs. 3250 2. It Alends Rs. 3500 to B at 10% p.a. and B ends the same sum to C at 11.5% p.a., then the gain of B (in Rs.) in a period of 3 years is: (@) 10750 (®) 11550 (©) 15750 (¢) 177.50 3. Avinash borrowed Rs. 6000 From Sanjay at simple Interest. After3 years, Sanjay got Rs. 300 more then what he had given to Avinash. What ‘was the rate ol interest per annum? (@) 2% (©) 5% © 8% (©) 10% 4. Rakesh took a loan for 6 years at the rate of 5% p.a. S.l. If the total interest paid was As. 1230, the principal was: (a) Rs. 4100 (b) Rs. 4920 (©) Rs. 5000 (@) Rs. 5300 5. Rs. 800 amounts to Rs. 920 in 3 years at simple interest. Ifthe interest rate is increased by.3%, tt would amount to how much? 10. WW. (a) As. 992 {o) As. 1112 (b) Rs. 1055 (0) Rs. 1182 ‘The simple interest On & SUM OF mong ig Rs, 48 for 4 years. The simpio inte, the same sum for 5 years at 4% wilh.” (@) Rs. 40 (b) As 48 (©) Rs. 50 (a) Rs. 60 A sum of money at simple interest ar to Rs. 2240 in 2 years and to Rs. 26 5 years. What is the principal amour (a) As. 1520 {o) Rs. 1880 (©) Rs. 2120 (2) Nore The simple interest on a certain sumat neve, at the rate of 5% p.a. for 8 years in Rs ‘At what rate of interest the same amoy interest can be received on the same suv after 5 years? @ 6% (c) 9% (b) 8% (d) 10% A sum of money was lent at simple interest at 11% p.a. for ag years ad 45 years respectively. Ifthe difference in ness for two period was Rs. 412.50, the sums (@) Rs. 3250 (b) Rs. 3500 (©) Rs. 3750 (0) Rs. 4250 Prabhat tooka certain amount as a loan to" a bank at the rate of 8% p.a. S.|, and 93 the same amount to Ashish as a loan at rate of 12% p.a. If at the end of 12 years made a profit of Rs. 320 in the deal, was the original amount. (a) Rs. 2000 (b) Rs. 3000 (c) Rs. 4000 (d) None of these Rahul borrowed Rs. 890 from Mr. Lal = pa. S|. for 3 years. He then added oo more money tothe borrowed sum 27° to Shobha for the same period at 14 104 | © Reasoning & Aptitude Scanned by CamScanner wADEe 13. Mr ate ot annum As 25 (@) As () 5 14, Theat of tse a) 15 n ¢€ 15. The s mone) prin amu at oé 16.4 sv ere rate of rte QUT SNANDOOD-Woradpresecom |v ‘ahul the whole transaction, hy : he add from his gi i @) Ps. 35 a (c) Rs. 80 AS Fs, 29.9 20 in ow much money aig (©) Rs. 55 @) Rs, 105 12. The simple intereston Rs, 1820 rom March rch 9, 1994 to May 21, 1 1 ‘: 204 at 75% tate will be (a) Rs. 29 (b) Rs (0) Rs. 27.30 on (d) Rs. 22.50 13. Mr. Roopchand finds that an increase in the ‘ Z rate of ered from 42%to sds per annum increases his yearly income by Rs. 25. His investment is: (@) Rs. 10,000 (b) As. 12,000 (©) Rs. 15,000 (d) Rs. 20,000 14. The rate at which a sum becomes four limes of itself in 15 years at S.I. will be: (a) 15% © Wh% {c) 20% (a) 25% 15. The simple interest accrued on 4 sum of money at the end of four years is Pinot its principal. What is the rate of Interest PO: annum? (@ 4% (o) 6% 46. A sum of money tl @months. Innowmary itself? (a) 6years3 months. (o) 7 years gmontns (c) B years 3 months {d) 9 years 6 months 1 (0) 5% (a) Data inadequate 1S Jes itself in 15 yea! is years wouit dovPle 17. The simple interes um, The number ot Scanned by CamScanner 18, 19. 21. ically nT eemng nce | 8 qual to the rate percent Percent per annum is: 1 Per annum. The rate 85 wis 2 65 @ 10 aan Of Rs. 10 is lent to be retuned in monthly installments of Re. 1 eacn, interest being simple. The rate of interests 1 @ 9t% (©) 10% (6) 11% @) 212% Hn The rate of simple interest ona sum of money is 6% pa for the frst 3 years, 8% 0. for the next 5 years and 10% p.a. for the period beyond 8 years Il the simple interest accrued by the sum for a total period of 10 years is Rs. 1560, what is the sum? (@) Rs. 1500 (p) Rs. 2000 (c) Rs. 3000 (@) Data inadequate A monthly installment of Rs. 180 is required to be paid for repayment of an interest free loan in 40 months. Ifit is decided to pay it'9 30 months, how much will be the montnty installment in rup9es? (@) © (b) 198 © 20 (@ 30 ‘a gum of Rs. 1850 is lent out into two parts, ‘oneal 8% and another one at 6% lithe totat acome AS. 106, the money let at annual i 8% 1s (a) Rs. 650 (©) Rs. /20 (o) Rs. (@) Rs. 900 gum Rs. 1550 was len! partly at 5% and ly at 8% pa. simple Interest Tre total Prrest eceived afiet 3 Yea's We fs. 300, Tne rato of the money Int 5% to tnat lent at 8%. yoursmahboob.wordpress.com @es €) 31-6 (0) 5.8 (0) 16.15 23. Wnat snouid be the least number of years 1" which the simple interest on Rs. 2600 at 2. 63% will De an exact number or 1usees? f@) 2 4 24, Aman invests an amount of Rs. 15860 in the names of his three sons A.B and Cin sucha way that they get the same amount after 2,3 and 4 years respectively If the rate of simple interest is 5%, then tne ratio ‘of amounts vested among. A, B and C will De: (@) 10:15:20 (0) 22:23:24 ) 6:43 (@) 2:3:4 25. Rs. 2189 are divided into three parts such mat tneir amounts after 1, 2 and 3 years respectively may be equal, the rate of simple interest being 4% p.a. in all cases. The smallest part is. (@) As 702 (©) Rs 756 ) 3 @s (o) Rs. 597 (d) Rs. 1083 Solutions 1. Ans. (c) 5 $1 =Rs| 2500x6x = |= s( xox) As.750. 2. Ans. (c) Gan ae 92011 $53 _ss00ut0x3) 100 100 = Rs. (12075 - 1050) = Rs. 157.50. 3. Ans (a) rate =( 4. Ans. (a) 100x300 $000 x3 rom 2%. 1230 x 100 Principal = Rs, ‘ ( 6x5 Scanned by CamScanner 5. 9. Ans. (a) Principal = RS 800, S1 =a -» Time = 3 years 100% 120 nate =( B00x3 } New rate = 8%, Princioal = Time 3 years. ‘oon fai Principal =F: incipal s( 7 Ans. (d) S.1. for 3 years = Rs. (2600 - 2240 = Rs. 360. Sl for 2 years = Rs. (22) Principal = Rs. (2240-240) =F 7 |. Ans. (b) Ans. (c) Let the sum be As. x. Then, 9.1 7 VxSx—- s (* SX oq TANKS = 41250 22x Seay 200 = oO yoursmahboob WAKARTESS.COM = 1841259 = +=3750 10. Ans. (d) Let the original amounts be As EX12%12_ xx8x12 100 “Foo ~ = 520 x. then, = 2000 “a =Rs 666.67 11. Ans. (d) (630+ x)x14x3 100 oF 830 x42 + 42 - 890 x36 = 9ga0 42x = 9320 - 4980 830x123 700 =93.90 Money added = Rs, 106. 12. Ans. (c) March, April, May 22 +30 + 21 =73 days 13, Ans. (a) a1 1 39, 1 225 6 100 = 2x= 20000 =9x= 10000. 14, Ans. (c) Let sum =x, Then, S:! (oar) = 20% ax — Rat 5x15 MADE EASY Scanned by CamScanner POUNA Interest | 497 18. Ans. (b) Let sum =, Then, Sia Then, St=2 Time = 4 years xd Rate ('s0x2 xt eon 16. Ans. (b) Let sum =x, Then, $1. = 2e and Time = 3 os Pate = (MEAP, 2) (2) x 31 3 Now, sum = x, S.l= x & Rate (2) = 7 years 9 months 17. Ans. (a) Let sum =x, Then, S..= Let time = n years and rate = n% 11 5 Then, N= 100xEx tet 400,104! = ne as te, ao tao gy 18. Ans. (d) Rs, 10+5.|. on Rs. 10 for 11 months = As.114 6.1. on Rel for (14243444 + 10) months Ag, 10+ Sil. on Re 1. for 110 months As. 11 +S1). on Re 1 for SS months S11. on Re 1 for 5 months = Re 1 49, Ans. (b) Let tne sum be Rs, x. Then __ —— > Reasoning & Aptitude | 107 0 \yagemsMmeunDOOb. wordpress.com a mars Ged, 8x8 , PION? 9 Bony de 10K: dete 0 i ‘0 ‘ 2 90:20 1526 404 t we 7Br= 156000 i & xno 2b. Ans. (b) 4 20. Ane. (¢) Lot those paris ier, y and (2i99 180 x40 = x30 oF r= 240 Then. ' rxted _yxexd : By. ane (a) "400 100, Lot the money lant at 8% be Rs. x. then, [eta9-frey)pxaxe , 28011, (1550 = 3)46%1 196 100 100 400 ‘ ar 2x +9300 =10600 oF x = 650 agro aaa) Byte _ (2180-39) xed Lot the sum al 8% be Fs. x. Then, 00 100 0x) x8 x3 OF Ady = 2189 x 12 8 zxbx , (8 eee 218912) or x= 800 %, y-( an Joo ‘Smallest part = Rs, 59/ ooo 23. Ans. (b) Compound Interest sttis(2600% 79 x 4 x1) aan 3 100 820 (r] Practice Lxercise | oho 3 *1), nn Is an exact number of rupees So T=3 1. Theamountof Re 7500 at compound! pa, anes) at 4% por alvium for 2 years, Lot tho arrounts ested be x,y, 7 respectively (a) fe. 7800 (b) Hs. e100 (eo) Hs 8112 (a) Hs BRE 1 THEM B YRIKB 2h 4 Te aod tod 2. Theultlerence between the com) and the sinale intorast ont a 8 sotto ye ha ewti for 2 years at 122.5% per anni 4 The sum is 108 | © Honwuniny & Aptitude wn” Scanned by CamScanner woressy VOUrsSmahboob.wordpress.com (@) Rs. 9000 (©) Rs.9500 (RS. 9200 () Rs. a600 3. The difference in com, POund interest, ee On @ Certain amount at ioe per ann mM at the end of the third year is . 620. What is the principal amount? (@) Rs. 40000 (b) Reno (b) Rs. 12000 (a) Rs. 20000 Ifthe compound interest on a certain sum at = 16, % for 3 years is Rs. 1270, the simple interest on the same sum at the same rate and for the same period is: (a) Rs, 1200 (b) Fs. 1165 (©) Rs. 1080 (@) Rs. 1220 5. The compound interest on a certain sum at 5% par annum for 2 years is Rs, 328. The simple interest for that sum at the same rate and for the same period will be: {a) As. 320 (b) Rs. 322 {c) Rs. 325 (d) Rs. 326 6. Whatis principal amount which earns Rs. 132 ‘as compound interest for the second year at 40% per annum? (a) Rs. 1000 (0) Rs. 1200 (c) Rs. 1320 (@) Rs. 1188 d interest 7. A sum of money at compoun amounts to Rs. 578.40 in 2 years and s Rs. 614.55 in3 years. The rate of interest P' annume is (@) 4% (o) 1 = % © 64% @ 3 8. A sum of money BMI in 3 yt 2 years and 10 Rs. compound interest The Sf 4260 fa rs 40 ea Tine aay Scanned by CamScanner * Simple Interest & Compound tnverest | 199 9. A sum of money at compound interest Amounts to thvice itz in 3 years. In mom Many years wil it be 9 times itself? @ 12 {o) 9 @s Or 10. In how many years will a sum of Rs. 800 at 10% per annum compounded semiannually become Hs, 926.107 1 @ 25 uf 2 Oy 1 © 25 (6) u To find out the total compound interest accrued on a sum of monay alter 5 years, which of the following informations given in the statements P and Q will be sufficient? P : The sum was Rs. 20000. Q: The total amount of simple interestn the ‘sum after 5 years was Rs. 4000. (a) Only Pis sufficient (6) Only Q is sufficient (©) Either P of Q is sufficient {d) Both P & Qare needed (0) Both P and Q are not sufficient W. 12, The least numberof complete years inwhich ‘sum of money put out at 20% co interest will be more than doubled 's: @ 3 (o) 4 @s 6 1 43, Aves increasearntally BY & thotitsheight. By how much wil nerease ater 2 years, it ? itetands today 64 om nian (a) 720m {b) 740m (@) 750m (d) 81cm interest on a sum for 2 years Jn 44. The compou ple intrest ont7e sa™1@ diference and ne simi 110 | (a) Rs. 48 (c) Rs 98.56 15. A sum of money becomes Fs. 19360 alter 3 years and Rs. 20070 after 6 yea’s on ‘compound interest. The sum is: (a) Rs. 8800 (b) Rs. 6890 (c) Rs. 8920 (a) As. 9040 ‘Acgum of Rs. 1100 was taken as a loan. This is to be repaid in two equal installments. If the rate of interest be 20% compounded ‘annually, then the value of each installments 7 (b) 66.56 (d) None of these 16, (@) Rs. 042 (0) Ps. 782 (c) Rs. 720 (@) Rs. 700 117. Te compound interest on Fs. 8000 at 15% per annurn for 2 years 4 months, compounded annually is: (a) Rs. 2980 (©) Rs. 3091 (© fs.3109 — (@) Rs. 3100 18. What annual payment will discharge a debt of Rs. 7620 due in 3 years at 163% per annum compound interest? (@) Rs,2540 ——(b) Rs. 3430 (©) Rs.3260 —(@) Re. 3380 19. A sum of money invested at compound interest amounts to Rs. 800 in 3 years and Fs. 840in 4 years. Whats the rate of interest per annum? (a) 2% (0) 4% © 5% (©) 10% 20. A sum of money becomes 8 times of itself in 3 years at compound interest. The rate of interest is: (@) 100% © 1% (b) 8% (@) Data inadequate Qogaa 110 | © Reasoning & Aptitude Scanned by CamScanner irpree: I BOob-wor 1. rdpress.com Solutions Ans. (c) ‘Amount = Rs| 7500%(1+ 4 ) 100, 26 26 =ns(7500% 55 a) =Rs Bt . Ans. (d) Let the sum be Rs. x. Then, : case] =e = (252 Ve 8*e Ans. (d) Let the sum be Rs. x. Then, | = 2103 _ 3x & 100 10 ey-n=( 3 ya00 = °° or x=20000 Hence, the principal amount is Fs yoursmahboob-Wworapressreom | 4, Ans. (c) - Let the sum be Rs, x. Then Che xx(i+ 2 L 3x100) [-* 34 -(G2-.)-2 216 216 27x ae =1270 270x214 or x “Ta 2160. Thus, the sum is Rs. 2160. 50 1 a si=Re(2160x52xax-4) = Rs, 1080. 5. Ans. (a) Let the sum be Rs. x. Then, : ciss( +55) -x 44te_\_ 4h -(Ge-*) a0 , 4% 2908 or x= “400 1 '3200x5x2) _ ps, 320. sare (to 6. Ans. (| (b) o vat the principal at tne end of Rs. x. then, 2210%1= 192 or x= 1320. 2 500 inal Now, let the origi Then, amount after 1 Ye" 1P xtoxt sa oD 0 MADE EASY Scanned by CamScanner 328x400 _ 3200. principal be RP ue ag 2820 or P= 2X10 1200, 7. Ans. (d) Interest on Rs. 578,40 tor 1 year = Fs. (61458 ~ 578.40) = Rs, 96.15 100x36.15 1 Rate = (se), 367%. 8. Ans. (a) Interest on Fs. 4624 for 1 year = Rs. (6083.50 -5290) = As. 793.50 100x793.50)4 _ 61, Rate =( Ree =65% 2 2 J = 4024 Now, {i F100 4717 X= = 4624 Fg IG 16 16 16.35) = Rs.4096. x+(s024x18 5) 9. Ans. (c) tet (By) -94(n) ac ale -( 4] (uty {using (01 Hence, n= 6 Years . Ans. (0) TO re = 5% paral YO: , cet time = 2n hall years = 9 YOON " Tneasoringé Aptitude | 7

You might also like